AFP Questions 9

अब Quizwiz के साथ अपने होमवर्क और परीक्षाओं को एस करें!

Schedule for Hep B vaccine

0, 2, 6 mo

Misleadingly low serum sodium can be caused by? (check one) A. Hyperglycemia B. Diuretic use C. Heart Failure D. Renal Disease

A decrease in serum sodium concentration does not always indicate a decrease in osmolality of body fluids. In cases of hyperglycemia, the main cause of the hyponatremia is the glucose-related increase in osmolality of extracellular fluid, followed by the movement of water from intracellular to extracellular fluid compartments and a subsequent loss of excessive extracellular fluid and electrolytes. The serum sodium concentration is also diminished in patients with hyperlipidemia or hyperproteinuria because of the volume occupied by the lipids or proteins. If the lipids or proteins are removed, the sodium concentration in the remaining plasma is found to be normal. No treatment is needed for these conditions.

A 15-year-old male is seen in the office for ankle pain. While playing basketball he jumped and landed on the lateral edge of his foot. He had immediate pain and did not continue playing, but was able to walk after the injury. On examination his right ankle has tenderness, swelling, and bruising over the anterior talofibular and calcaneofibular ligaments. There is no bony tenderness.Which one of the following would be most appropriate at this point? (check one) A. Taping the ankle for future sports participation B. An elastic compression wrap C. A lace-up ankle support D. A radiograph of the ankle E. A below the knee cast

A lace-up ankle support reduces pain and recovery time after an ankle sprain (SOR B). The Ottawa Rules state that radiography is required only if there is pain in the malleolar or midfoot zone and either bony tenderness over an area of potential fracture (i.e., distal fibula or tibia, lateral or medial malleolus, base of the fifth metatarsal, or navicular bone) or an inability to bear weight immediately after the injury and when evaluated by a physician. This patient did not have those findings, and therefore would not need a radiograph (SOR A). A cast is not necessary for an ankle sprain. An elastic compression wrap alone is not as effective as a lace-up support. Taping of the ankle for future sports participation can reduce the risk of ankle sprains during sports, but would not be appropriate for an acute injury.

A neurologically intact 77-year-old female presents with severe low back pain following a fall 2 days ago that caused her to land on her buttocks. A radiograph of her lower spine shows a compression fracture of L3 with a loss of about 50% of the vertebral body height.Which one of the following is most appropriate at this point? (check one) A. Referral for kyphoplasty B. Referral for vertebroplasty C. Back bracing D. Bed rest E. Calcitonin-salmon (Miacalcin)

A number of measures for managing spinal compression fractures have been evaluated. The evidence for recommending kyphoplasty is weak, and the evidence for recommending against vertebroplasty is strong. The data on bracing is inconclusive, as is the recommendation for bed rest. Calcitonin has been shown to reduce the incidence of recurrent fractures and may be useful in the relief of pain.

When compared with an occiput anterior fetal position, a persistent occiput posterior fetal position is less likely to result in? (check one) A. Spontaneous vaginal delivery B. Assisted vaginal delivery C. Cesarean delivery D. A third or fourth degree perineal laceration E. Excessive maternal blood loss

A persistent occiput posterior position is associated with a higher risk of cesarean delivery and assisted vaginal delivery, and a lower chance of spontaneous vaginal delivery. Assisted vaginal deliveries are associated with a higher rate of third-and fourth-degree perineal lacerations and postpartum hemorrhage.

A 17-year-old football player presents to your office on a Monday with a right knee injury. He injured the knee in Friday night's game when an opposing player fell against the knee from the front while the patient had his right foot planted. He was unable to bear weight after the injury, and noted immediate swelling of the knee.A positive result with which one of the following would indicate an anterior cruciate ligament tear? (check one) A. Ballottement test B. Lachman test C. McMurray test D. Posterior drawer test E. Thessaly test

A positive Lachman test indicates that the anterior cruciate ligament may be torn. The posterior drawer test evaluates posterior cruciate ligament stability. The McMurray and Thessaly assessments test for meniscal tears. The ballottement test is for detecting intra-articular knee effusion.

Which one of the following medications used to treat psychiatric disorders is associated with an increased risk of agranulocytosis? (check one) A. Carbamazepine (Tegretol) B. Lithium C. Aripiprazole (Abilify) D. Olanzapine (Zyprexa) E. Imipramine (Tofranil)

A. Carbamazepine (Tegretol) People taking carbamazepine have a five-to eightfold increased risk of developing agranulocytosis. Baseline values including a CBC, serum electrolytes, and liver enzymes should be obtained before the drug is started, and the patient should be monitored with periodic hematologic testing. The other medications listed are not associated with agranulocytosis. Aripiprazole and olanzapine carry black box warnings for an increased risk of death in the elderly. Lithium is associated with lithium toxicity and thyroid dysfunction. Imipramine carries a warning for cardiac toxicity, and EKG monitoring is recommended.

A 72-year-old male is brought to your office by a friend because of increasing confusion, irritability, and difficulty walking. This began shortly after the patient's car broke down in a rural area and he had to walk a mile to get to a phone and call the friend. The temperature outdoors has been near 100°F.On examination the patient has a rectal temperature of 39.5°C (103.1°F), a pulse rate of 110 beats/min, and a blood pressure of 100/60 mm Hg. His shirt is still damp with sweat.Which one of this patient's findings indicates that he has heatstroke rather than heat exhaustion? (check one) A. Confusion B. Sweating C. His temperature D. His heart rate E. His blood pressure

A. Confusion A Heat exhaustion and heatstroke are both on the continuum of heat-related illness. Heatstroke is a much more severe condition than heat exhaustion. Evidence of central nervous system dysfunction is evidence of heatstroke rather than heat exhaustion, even if other symptoms are not severe and point to heat exhaustion. Heatstroke is a medical emergency.

A 32-year-old male presents to an urgent care center with a 2-day history of left calf pain and swelling, which started gradually a few hours after he played tennis. He remembers that he "tweaked" his calf on a serve late in the match but was able to continue playing. He has no history of prior medical problems, and no recent surgery or immobilization.On examination his left calf appears slightly erythematous and swollen from the mid-calf to the ankle, with 1+ pitting over the lower leg. There is no venous distention. The left calf is 3 cm greater in circumference than the right calf. He has pain with dorsiflexion, and there is an area of tenderness in the medial calf.Which one of the following is the most appropriate next step in ruling out deep vein thrombosis in this patient? (check one) A. D-dimer B. Ultrasonography C. Venography D. Impedance plethysmography

A. D-dimer A number of pretest probability scoring systems are available for assessing venous thromboembolism (VTE), which includes deep vein thrombosis (DVT) and pulmonary embolism. Although the Wells clinical prediction rule is widely used, other tools such as the Hamilton score and the AMUSE (Amsterdam Maastricht Utrecht Study on thromboEmbolism) score are also available. The Wells rule divides patients suspected of having a DVT into low, intermediate, and high-risk categories, with a 5%, 17%, and 53% prevalence of DVT, respectively. This patient has a Wells score of 0 (+1 for calf circumference increase >3 cm, +1 for pitting edema, -2 for a likely alternative diagnosis of gastrocnemius strain) and is therefore at low risk. A negative D-dimer assay has a high negative predictive value for DVT, so the diagnosis can be ruled out in a patient who has a low pretest probability and a negative D-dimer result. A negative D-dimer assay does not rule out DVT in a patient with a moderate to high pretest probability (SOR C).

What is the specific antidote used to treat methanol poisoning? (check one) A. Ethanol B. Haloperidol C. Lorazepam (Ativan) D. Naloxone E. Thiamine

A. Ethanol The current management of methanol intoxication, depending on its severity, includes ethanol administration to inhibit the metabolism of methanol, hemodialysis to remove alcohol and its toxins, and vigorous management of metabolic acidosis with bicarbonate therapy. Ethanol is a competitive inhibitor of toxin metabolism and slows the formation of toxic metabolites, formaldehyde, and formic acid from methanol, permitting these products to be disposed of by ordinary metabolic or excretory pathways. It has a similar effect in ethylene glycol poisoning, slowing the formation of glycoaldehyde and glycolic, glyoxylic, and oxalic acids.

Which one of the following is most characteristic of the pain associated with acute pericarditis? (check one) A. Improvement when sitting up and leaning forward B. Improvement when lying supine C. Worsening with the Valsalva maneuver D. Radiation to the right scapula E. Radiation to both arms

A. Improvement when sitting up and leaning forward While there is substantial overlap in the signs, symptoms, and physical findings for the various etiologies of chest pain, a good history and physical examination can help determine which patients require immediate further evaluation for a potentially serious cause. The chest pain associated with pericarditis is typically pleuritic, and is worse with inspiration or in positions that put traction on the pleuropericardial tissues, such as lying supine. Patients with acute pericarditis typically get relief or improvement when there is less tension on the pericardium, such as when sitting and leaning forward. This position brings the heart closer to the anterior chest wall, which incidentally is the best position for hearing the pericardial friction rub associated with acute pericarditis.Radiation of chest pain to both arms should raise concerns about myocardial ischemia or infarction. Radiation to the right scapula is sometimes seen with cholelithiasis. Worsening pain with the Valsalva maneuver is nonspecific and is of no particular diagnostic value.

A 64-year-old male with a previous history of hypertension and atrial fibrillation presents with an acute onset of ataxia, headache, mild confusion, and restlessness. His only current medications are lisinopril (Prinivil, Zestril) and warfarin (Coumadin). On examination his blood pressure is 160/100 mm Hg, pulse rate 86 beats/min, respirations 12/min, and temperature 36.7°C (98.1°F). A CBC, serum electrolyte levels, and cardiac enzyme levels are normal. His INR is 1.1. Noncontrast CT shows a cerebellar hemorrhage with a hematoma volume of 50 mL.Which one of the following should be performed urgently? (check one) A. Neurosurgical consultation for posterior cerebellar hematoma decompression B. A reduction in blood pressure to 140/90 mm Hg C. Administration of vitamin K, 10 mg intravenously D. Administration of mannitol (Osmitrol), 0.5-1.0 mg/kg intravenously E. Induction of hypothermia to achieve a body temperature of 34.4°C (94.0°F)

A. Neurosurgical consultation for posterior cerebellar hematoma decompression Aggressive neurosurgical intervention is not indicated to evacuate clots in patients with intracerebral hemorrhage except in those with a cerebellar hemorrhage, which is always an indication for neurosurgical consultation. Guidelines have been developed by the American Heart Association for lowering blood pressure in patients with a systolic blood pressure >180 mm Hg, or a mean arterial pressure >130 mm Hg. The use of various forms of osmotherapy, including mannitol, to prevent the development of cerebral edema has not been shown to improve outcomes. The data regarding hypothermia induction is unclear. Patients with an INR >1.5 should receive therapy to replace vitamin K-dependent factors and have their warfarin withheld.

A 3-year-old female is brought to your office for evaluation of mild intoeing. The child's patellae face forward, and her feet point slightly inward.Which one of the following would be most appropriate? (check one) A. Reassurance and continued observation B. Foot stretching exercises C. Orthotics D. Night splints E. Surgery

A. Reassurance and continued observation Intoeing, as described in this patient, is usually caused by internal tibial torsion. This problem is believed to be caused by sleeping in the prone position and sitting on the feet. In 90% of cases internal tibial torsion gradually resolves without intervention by the age of 8. Avoiding sleeping in a prone position enhances resolution of the problem. Night splints, orthotics, and shoe wedges are ineffective. Surgery (osteotomy) has been associated with a high complication rate, and is therefore not recommended in mild cases before the age of 8.

A 50-year-old female reports a 1-month history of pain in her wrists. She does not recall any injury. On examination both wrists are warm but not red, feel boggy on palpation, and lack 30° of both flexion and extension. No other joints are affected. She feels fatigued and unwell, but attributes this to her busy schedule. Radiographs of the wrists are normal. Laboratory findings are unremarkable except for a mildly elevated erythrocyte sedimentation rate and a negative rheumatoid factor.Which one of the following is the most likely diagnosis? (check one) A. Rheumatoid arthritis B. Osteoarthritis C. Inapparent injury D. Fibromyalgia E. Lyme disease

A. Rheumatoid arthritis Rheumatoid arthritis is most often symmetric at presentation and particularly affects the wrists and other extremity joints that have a high ratio of synovium to articular cartilage. Rheumatoid factor is often negative in the early months of the disease, although it may be positive later. Radiographs and laboratory tests are helpful, but the diagnosis is primarily clinical. Osteoarthritis of the wrists usually involves the carpal-metacarpal joint of the thumb primarily, and the joint would be red if there were an injury. Fibromyalgia usually involves the soft tissue of the trunk, and there is no evidence of inflammation. Lyme disease can cause a variety of joint diseases, but not chronic symmetric arthritis.

During a routine office visit, a 65-year-old female asks if she should be screened for carotid artery stenosis. The patient has a history of controlled hypertension and hypercholesterolemia, and a family history of stroke. Physical examination of the carotid artery is normal and the patient is asymptomatic.Which one of the following is consistent with U.S. Preventive Services Task Force and American Heart Association recommendations regarding carotid artery ultrasonography for this patient? (check one) A. She does not need screening ultrasonography at this time B. She should have one-time screening ultrasonography now C. She should have routine screening ultrasonography now and every 5 years D. She should have routine screening ultrasonography now and every 10 years

A. She does not need screening ultrasonography at this time The U.S. Preventive Services Task Force and the American Heart Association/American Stroke Association recommend not performing carotid artery screening with ultrasonography or other screening tests in patients without neurologic symptoms because the harms outweigh the benefits. In the general population, screening tests for carotid artery stenosis would result in more false-positive results than true-positive results. This would lead to surgical procedures that are not indicated or to confirmatory angiography. As a result of these procedures, some patients would suffer serious harms such as death, stroke, or myocardial infarction, which outweigh the potential benefit surgical treatment may have in preventing stroke.

The diagnosis of Osgood-Schlatter disease (osteochondritis of the tibial tubercle apophysis) is best made on the basis of findings from: (check one) A. the history and examination B. evaluation by an orthopedic specialist C. radiographs D. ultrasonography E. MRI

A. the history and examination Osgood-Schlatter disease is an inflammatory condition that is a common cause of knee pain in children and adolescents. The diagnosis is usually based on clinical findings, although radiographs may be necessary to rule out fractures or other problems if findings are not typical. MRI, ultrasonography, and orthopedic referral are not usually needed. The problem is typically self-limited and responds to activity modification, over-the-counter analgesics, stretching, and physical therapy.

A 55-year-old male has New York Heart Association class III chronic systolic heart failure due to hypertensive cardiomyopathy. Which one of the following is CONTRAINDICATED in this patient? (check one) A. Carvedilol (Coreg) B. Digoxin C. Ramipril (Altace) D. Spironolactone (Aldactone) E. Verapamil (Calan)

ACE inhibitors and β-blockers improve mortality in heart failure (HF). Digoxin and furosemide improve symptoms and reduce hospitalizations in systolic HF, and furosemide may decrease mortality. Spironolactone, an aldosterone antagonist, reduces all-cause mortality and improves ejection fractions in systolic HF. Verapamil, due to its negative inotropic effect, is associated with worsening heart failure and an increased risk of adverse cardiovascular events.

Which one of the following is a preferred first-line agent for managing hypertension in patients with stable coronary artery disease? (check one) A. A thiazide diuretic B. An angiotensin receptor blocker C. A β-blocker D. A long-acting calcium channel blocker E. A long-acting nitrate

American Heart Association guidelines recommend treating hypertension in patients with stable heart failure with ACE inhibitors and/or β-blockers. Other agents, such as thiazide diuretics or calcium channel blockers, can be added if needed to achieve blood pressure goals (SOR B). β-Blockers with intrinsic sympathomimetic activity should be avoided, as they increase myocardial oxygen demand.While thiazide diuretics are often a first choice for uncomplicated hypertension, this is not the case for patients with coronary artery disease. Long-acting calcium channel blockers may be used in patients who do not tolerate β-blockers, but short-acting calcium channel blockers should be avoided because they increase mortality. ACE inhibitors are recommended as antihypertensive agents in patients already on β-blocker therapy (especially following myocardial infarction), in diabetics, and in patients with left ventricular dysfunction. Although angiotensin receptor blockers have indications similar to those of ACE inhibitors, the American Heart Association recommends using them only in patients who do not tolerate ACE inhibitors. Long-acting nitrates are used for their anti-anginal properties and have no role in the management of hypertension.

A 49-year-old uninsured female with diabetes mellitus presents with painful burning of her feet, particularly at night. She has tried ibuprofen and acetaminophen without relief. Her last hemoglobin A1c was 7.1%. Her medications include metformin (Glucophage), glipizide (Glucotrol), lisinopril (Prinivil, Zestril), and lovastatin (Mevacor).Which one of the following would be the best choice to treat her foot pain? (check one) A. Amitriptyline B. Topiramate (Topamax) C. Fluoxetine (Prozac) D. Lamotrigine (Lamictal)

Amitrip First-line treatment for diabetic peripheral neuropathy, according to the American Diabetes Association, is tricyclic antidepressants. Anticonvulsants are second line and opioids are third line. Many medications have been found to be effective, including the tricyclics, duloxetine, pregabalin, oxycodone, and tramadol (SOR A).Among the tricyclics, amitriptyline, imipramine, and nortriptyline have been found to be the most effective (SOR A). For an uninsured patient, the tricyclics are also the most affordable.

Which one of the following is most likely to cause hypoglycemia in elderly patients? (check one) A. Metformin (Glucophage) B. Pioglitazone (Actos) C. Glipizide (Glucotrol) D. Sitagliptin (Januvia) E. Glyburide (DiaBeta)

Among the oral antiglycemic drugs, the sulfonylurea agents are the most likely to cause hypoglycemia, and glyburide is more likely to cause hypoglycemia than glipizide. Glyburide should rarely be used in the elderly.

Which one of the following is considered first-line therapy for mild to moderate Alzheimer's disease? (check one) A. Donepezil (Aricept) B. Memantine (Namenda) C. Selegiline (Eldepryl) D. Risperidone (Risperdal) E. Ginkgo biloba

Anticholinesterase inhibitors such as donepezil are considered first-line therapy for patients with mild to moderate Alzheimer's disease (SOR A). Memantine is an NMDA receptor antagonist and is often used in combination with anticholinesterase inhibitors for moderate to severe Alzheimer's disease, but it has not been shown to be effective as a single agent for patients with mild to moderate disease. There is not enough evidence to support the use of selegiline, a monoamine oxidase type B inhibitor, in the treatment of Alzheimer's disease. Risperidone and other antipsychotic medications are not approved by the Food and Drug Administration for treatment of Alzheimer's disease, but can sometimes be helpful in controlling associated behavioral symptoms. Studies of ginkgo biloba extract have not shown a consistent, clinically significant benefit in persons with Alzheimer's disease.

A 56-year-old African-American male with long-standing hypertension and a 30-pack-year smoking history has a 2-day history of dyspnea on exertion. A physical examination is unremarkable except for rare crackles at the bases of the lungs.Which one of the following serologic tests would be most helpful for detecting left ventricular dysfunction? (check one) A. B-type natriuretic peptide B. Troponin T C. C-reactive protein (CRP D. D-dime E. Cardiac interleukin-2

B-type natriuretic peptide (BNP) is a 32-amino acid polypeptide secreted from the cardiac ventricles in response to ventricular volume expansion and pressure overload. The major source of BNP is the cardiac ventricles, and because of the minimal presence of BNP in storage granules, its release is directly proportional to ventricular dysfunction. A BNP test is simple and time efficient, and reliably predicts the presence or absence of left ventricular dysfunction on an echocardiogram.

A 3-year-old male is brought to your office by his father for evaluation of 5 days of knee pain and fever up to 101.6°F. There was no known trauma preceding these symptoms. The pain and fever respond well to oral acetaminophen but continue to recur 4 hours after each dose.On examination the child appears well and is afebrile. He had a dose of acetaminophen about 2 hours ago. There are no signs of upper respiratory infection. Examination of the knee reveals no redness, warmth, or swelling, and you see no other skin changes. He has full range of motion of both the knee and hip without pain. You note tenderness to firm palpation of the proximal tibia. He is able to bear weight and walk but refuses to jump due to anticipation of pain in his knee. Plain films of the knee are normal.The next step in the evaluation of this patient should include which one of the following? (check one) A. Close monitoring at home B. A CBC, a C-reactive protein level, and an erythrocyte sedimentation rate C. Ultrasonography of the hip D. Knee joint aspiration E. MRI of the knee

B. A CBC, a C-reactive protein level, and an erythrocyte sedimentation rate Joint pain in the presence of fever with no apparent source indicates a possible infection, malignancy, or rheumatologic condition and requires further workup. Laboratory evaluation, including a CBC, a C-reactive protein level, and an erythrocyte sedimentation rate can help assess for these conditions, even though none of the tests is sufficiently sensitive to rule out these diseases, and they are not specific to a single disease entity. Knee joint aspiration would be indicated to rule out septic arthritis in the presence of a joint effusion. If the hip were painful or had decreased range of motion, then ultrasonography could help identify a hip joint effusion, which would need to be aspirated. MRI may be needed in this patient, but it would likely require sedation and thus is more invasive. Starting with laboratory work is a good first step toward identifying the source of his pain and fever.

A 48-year-old female presents as a new patient to your office. She has not seen a physician for several years and her medical history is unknown. Her BMI is 24.4 kg/m2 and she is not taking any medication. Her blood pressure is 172/110 mm Hg in the left arm sitting and 176/114 mm Hg in the right arm sitting; her cardiovascular examination is otherwise unremarkable. A baseline metabolic panel reveals a creatinine level of 0.68 mg/dL (N 0.6-1.1) and a potassium level of 3.3 mEq/L (N 3.5-5.5).If the patient's hypertension should prove refractory to treatment, which one of the following tests is most likely to reveal the cause of her secondary hypertension? (check one) A. A 24-hour urine catecholamine level B. A plasma aldosterone/renin ratio C. MRA of the renal arteries D. Echocardiography E. A sleep study (polysomnography)

B. A plasma aldosterone/renin ratio Primary hyperaldosteronism is the most common cause of secondary hypertension in the middle-aged population, and can be diagnosed from a renin/aldosterone ratio. This diagnosis is further suggested by the finding of hypokalemia, which suggests hyperaldosteronism even though it is not present in the majority of cases.An echocardiogram would help make a diagnosis of coarctation of the aorta, but this is more common in younger patients. Renal MRA may demonstrate renal artery stenosis, but this condition is more common in older patients. Sleep apnea is increasing in prevalence along with the rise in obesity, but it is not suggested by this case. A 24-hour urine catecholamine test is used to diagnose pheochromocytoma, which is not suggested by this patient's findings. Pheochromocytoma is also less common than aldosteronism (SOR C).

You prescribe enalapril (Vasotec) for a 68-year-old male with heart failure. At a follow-up visit 6 weeks later the patient's serum creatinine level is 2.5 mg/dL (N 0.6-1.5) and his serum potassium level is 5.7 mEq/L (N 3.4-4.8). His baseline values were normal.Which one of the following is a side effect of ACE inhibitors that is the most likely cause of these changes in renal function? (check one) A. Toxicity to the proximal renal tubules B. Impaired autoregulation of glomerular blood flow C. Microangiopathic arteriolar thrombosis D. Rhabdomyolysis E. Interstitial nephritis

B. Impaired autoregulation of glomerular blood flow Blood flow to the kidney is autoregulated so as to sustain pressure within the glomerulus. This is influenced by angiotensin II-related vasoconstriction. ACE inhibitors can impair the kidney's autoregulatory function, resulting in a decreased glomerular filtration rate and possibly acute renal injury. This is usually reversible if it is recognized and the offending agent stopped. NSAIDs can exert a similar effect, but they can also cause glomerulonephritis and interstitial nephritis. Statins, haloperidol, and drugs of abuse (cocaine, heroin) can cause rhabdomyolysis with the release of myoglobin, which causes acute renal injury. Thrombotic microangiopathy is a rare mechanism of injury to the kidney, and may be caused by clopidogrel, quinine, or certain chemotherapeutic agents.

A 60-year-old male smoker has lung cancer, and a life expectancy of 4-6 months. His wife is concerned about his state of mind and requests medication for him. His cancer-related pain is generally controlled.When evaluating the patient, which one of the following features would be more characteristic of depression as opposed to a grief reaction? (check one) A. Insomnia B. Loss of interest or pleasure in all activities C. Feelings of guilt D. Thoughts of wanting to die E. Psychomotor agitation

B. Loss of interest or pleasure in all activities While there is significant overlap in the symptoms of each condition, there are some signs and symptoms that help the family physician determine whether a terminally ill patient is experiencing grief or has major depression. This distinction is important because the terminally ill patient with depression would likely benefit from antidepressant medication, whereas a patient with end-of-life grief is generally best managed without psychotherapeutic medications. The key clinical feature in distinguishing the two conditions is in the pervasiveness of symptoms in depression, particularly the loss of pleasure or interest in all activities. Episodic feelings of guilt, anxiety, and helplessness, and even thoughts of wanting to die can and do occur with grief reactions, but these feelings are not constant and over time the symptoms gradually wane. Terminally ill patients with major depression feel helplessly hopeless all the time, but they often respond to and significantly benefit from antidepressant medication (SOR A).

A 54-year-old female presents with a complaint of dizziness. Two days ago, while riding in a friend's car and trying to read a book, she experienced sudden extreme nausea and a "spinning" feeling that lasted for 20 minutes. She also had a headache that mainly felt like a fullness in the area around her left ear. Since then she has had only mild dizziness when she moves her head too quickly. She recalls experiencing these symptoms on two other occasions but cannot remember the circumstances, although she thinks one episode may have been related to having had too much caffeine. A review of systems is positive for a humming in her ears over the last few years. On examination both ears appear normal. Mild horizontal nystagmus can be seen on movement of the head to the left. Audiograms are normal in the right ear, with a low-frequency hearing loss on the left.Which one of the following is the most likely diagnosis? (check one) A. Motion sickness B. Meniere's disease C. Vestibular migraine D. Benign positional vertigo

B. Meniere's disease This patient's symptoms are compatible with Meniere's disease, which is characterized by multiple episodes of vertigo lasting for 20-120 minutes, accompanied by a fluctuating hearing loss, tinnitus, and a sense of aural fullness. Audiograms will reveal a low-frequency hearing loss with an upsloping curve, which can become flattened over the years. Most patients develop unilateral symptoms, and many patients will develop bilateral disease many years after the onset of the unilateral symptoms. Multiple studies have reported the rate of bilateral Meniere's disease to be as high as 50% many years after the initial diagnosis. Motion sickness is a common cause of nausea, but the nausea usually does not come on suddenly and is not as pronounced as with Meniere's disease. A vestibular migraine can present like a sudden Meniere's disease attack but in this patient the audiograms, tinnitus, and aural fullness suggest Meniere's disease. Benign positional vertigo is very common, and hearing loss could be an incidental finding. However, the most common form of age-related hearing loss is seen at the higher frequencies. Positional vertigo like this patient has is common between attacks of Meniere's disease. There is often a family history of Meniere's disease, and there is frequently an association with allergies. The condition can also get worse with caffeine use. Even though the diagnosis is clinical, MRI and blood tests are recommended to rule out other conditions that may be putting pressure on the endolymphatic system and thus causing the symptoms.

An 87-year-old female is brought to the emergency department after losing consciousness at the dinner table. Her history indicates recent unintentional weight loss. Further evaluation ultimately reveals a large mass at the head of the pancreas and extensive metastasis to numerous organs, including the brain. Her life expectancy is estimated to be 2-3 weeks. The patient chooses to receive hospice care but becomes very depressed.Which one of the following would be best for improving her depression? (check one) A. Electroconvulsive therapy B. Methylphenidate (Ritalin) C. Mirtazapine (Remeron) D. Fluoxetine (Prozac) E. Nortriptyline (Pamelor)

B. Methylphenidate (Ritalin) There is good evidence that psychostimulants reduce symptoms of depression within days, making methylphenidate a good choice for this patient (SOR B). Electroconvulsive therapy is contraindicated due to her brain lesions. Mirtazapine, fluoxetine, and nortriptyline all take at least 3-4 weeks to have any antidepressant effects, and would not be appropriate given the patient's life expectancy (SOR B).

A 17-year-old female presents to your office with anterior knee pain. She tells you she recently started a running program. She says the pain is worse running down hills, and is vaguely localized just medial to the patella. Examination of the knee shows no effusion or instability, and there is no joint-line pain or patellar tenderness. McMurray's maneuver is negative. Plain radiographs of the knee appear normal.Which one of the following would be most appropriate at this point? (check one) A. MRI of the knee B. Modification of her running program and a quadriceps and hip strengthening program C. Static stretching of the quadriceps and hamstrings prior to running D. A corticosteroid injection in the area of the pes anserine bursa

B. Modification of her running program and a quadriceps and hip strengthening program This patient is suffering from patellofemoral pain syndrome, which causes anterior knee pain that is worse with running downhill. The examination is often normal, although there may be apprehension when the knee is extended with pressure over the patella and the patella will sometimes track laterally. Patellofemoral pain syndrome can be treated with exercises to strengthen the quadriceps and hips, and by using a knee sleeve with a doughnut-type cushion that the patella fits into. Static stretching would not address the problem. MRI would be indicated if there were joint-line pain or an unstable knee. Pes anserine bursitis usually causes pain and tenderness medially, below the joint line.

A 52-year-old white male presents for a health maintenance visit. The patient has mild osteoarthritis but is otherwise healthy. He lives at home with his wife. He drinks approximately 2 beers a week and does not smoke. He takes a multivitamin, but no other medications.What is the recommendation for immunizing this patient with pneumococcal polyvalent-23 vaccine (Pneumovax)? (check one) A. One dose now B. One dose after age 65 C. One dose now, and again after age 65 D. One dose now, in 5 years, and again after age 65 E. No vaccination unless he develops an immunocompromising disease

B. One dose after age 65 For a healthy nonsmoker with no chronic disease who is not in a high-risk group, pneumococcal vaccine is recommended once at age 65, or as soon afterward as possible. Persons that should be immunized before age 65 include patients with chronic lung disease, cardiovascular disease, diabetes mellitus, chronic liver disease, cerebrospinal fluid leaks, cochlear implants, immunocompromising conditions, or asplenia, and residents of nursing homes and long-term care facilities. The Advisory Committee on Immunization Practices of the CDC updated the recommendations for pneumococcal vaccination in 2011 to include immunization for persons age 50-64 in the following categories: Alaska Natives, Native Americans living in areas of increased risk, persons with asthma, and smokers.

A 32-year-old white primigravida has a stillbirth at 33 weeks gestation. Which one of the following is the most likely cause? (check one) A. Infection B. Placental disease C. A fetal structural disorder D. A hypertensive disorder

B. Placental disease Stillbirth is defined as fetal death occurring at or after 20 weeks gestation, and affects approximately 1 in 160 pregnancies in the United States. A large study of stillbirths from 2006 to 2008 tried to establish a cause in 663 cases, and a probable or possible cause was identified in approximately 75% of these. While there were some significant ethnic differences, placental abnormalities and obstetric complications were the largest category of causes in white women, and this was even more true after 32 weeks gestation. Other important causes included infection and fetal defects. More than one cause was found in one-third of cases.

A 12-month-old male is brought to your office for a routine checkup and immunizations. He has not received medical care since his 4-month well child visit and has had no immunizations since that time.Which one of the following vaccines is NOT indicated for this patient? (check one) A. Varicella vaccine B. Rotavirus vaccine C. Hepatitis B vaccine D. MMR vaccine

B. Rotavirus vaccine In general, when young children are found to be behind schedule in receiving recommended immunizations, catch-up immunization is important. However, the rotavirus series should not be started past 15 weeks of age, or continued past 8 months of age. This child should have received hepatitis B vaccine at 6 months of age, and should be given a catch-up dose. The MMR and varicella vaccines are recommended at the 12-month visit.

A 78-year-old male is on dual antiplatelet therapy (aspirin and clopidogrel) as a result of a stroke 6 months ago. He recently underwent coronary angiography, and his cardiologist has scheduled coronary artery bypass surgery for a week from today.Which one of the following is recommended with regard to his antiplatelet therapy? (check one) A. Stopping only aspirin 5 days before surgery B. Stopping only clopidogrel 5 days before surgery C. Stopping both aspirin and clopidogrel 5 days before surgery D. Continuing both aspirin and clopidogrel

B. Stopping only clopidogrel 5 days before surgery Patients receiving dual antiplatelet therapy who require bypass surgery should continue taking aspirin. Clopidogrel or prasugrel should be stopped 5 days before the surgery due to the increased risk of major bleeding during surgery.

Which one of the following activities is most likely to be impaired in early dementia? (check one) A. Dressing B. Eating C. Toileting D. Grooming E. Cooking

Basic activities of daily living, such as dressing, eating, toileting, and grooming, are generally intact in early dementia. In contrast, instrumental activities of daily living, such as managing money and medications, shopping, cooking, housekeeping, and transportation, which often require calculation or planning, are frequently impaired in early dementia.

A 67-year-old female hospitalized with pneumonia develops the rapid onset of dyspnea, pleuritic chest pain, tachypnea, and hypoxemia not responding to oxygen and requiring intubation. A physical examination is notable for rales throughout both lung fields with no peripheral edema noted. A chest radiograph shows bilateral pulmonary infiltrates. Her BNP level is 90 ng/L.Which one of the following is the most likely reason for her worsening clinical situation? (check one) A. Heart failure B. Hypersensitivity pneumonitis C. Acute respiratory distress syndrome D. Pulmonary embolus E. Pneumothorax

C. Acute respiratory distress syndrome This patient demonstrates classic findings for acute respiratory distress syndrome (ARDS). In many cases ARDS must be differentiated from heart failure. Heart failure is characterized by fluid overload (edema), jugular venous distention, a third heart sound, an elevated BNP level, and a salutary response to diuretics. A BNP level <100 pg/mL can help rule out heart failure (SOR A). In addition, a patient with ARDS would not have signs of left atrial hypertension and overt volume overload. Hypersensitivity pneumonitis is usually preceded by exposure to an inciting organic antigen such as bird feathers, mold, or dust. Pulmonary embolus, while certainly in the differential, is unlikely to cause such dramatic radiographic findings. Pneumothorax would be seen on the chest radiograph.

Which one of the following is the basis for the most effective method of natural family planning? (check one) A. Calendar calculation B. Basal body temperature charting C. Cervical mucus monitoring D. Monitoring for urine estrogen metabolites E. Coitus interruptus (withdrawal)

C. Cervical mucus monitoring Natural family planning (NFP) is a potentially effective method for contraception and for determining the time of ovulation for purposes of conception. While the contraceptive effectiveness of the different NFP methods varies significantly, the success rates for typical use are as high as 92%-98% (SOR B). Monitoring the presence and consistency of cervical mucus production allows for the determination of both the beginning and end of a woman's most fertile period. Some NFP methods use cervical mucus secretion as the sole basis for determining fertility. The symptothermal method also incorporates calendar calculations, basal body temperature measurement, and ovulation-related symptoms as a complement to the cervical mucus component. The Marquette Model incorporates cervical mucus and basal body temperature charting with electronic monitoring of urine estrogen and LH metabolites to provide additional information to determine when ovulation has occurred.

A 25-year-old white male truck driver presents with a 1-day history of throbbing rectal pain. Your examination shows a large thrombosed external hemorrhoid.Which one of the following is the preferred initial treatment for this patient? (check one) A. Infrared coagulation B. Rubber band ligation of the hemorrhoid C. Elliptical excision of the thrombosed hemorrhoid D. Stool softeners and a topical analgesic/hydrocortisone cream

C. Elliptical excision of the thrombosed hemorrhoid The appropriate management of a thrombosed hemorrhoid presenting within 72 hours of the onset of symptoms is elliptical excision of the hemorrhoid and overlying skin under local anesthesia, such as 0.5% bupivacaine hydrochloride in 1:200,000 epinephrine, infiltrated slowly with a 27-gauge needle.Incision and clot removal may provide inadequate drainage, resulting in rehemorrhage and clot reaccumulation. Most thrombosed hemorrhoids contain multilocular clots that may not be accessible through a simple incision. Rubber band ligation is an excellent technique for management of internal hemorrhoids, and infrared coagulation is also used for this purpose. Banding an external hemorrhoid would cause exquisite pain.If the pain is already subsiding or more time has elapsed, and if there is no necrosis or ulceration, measures such as sitz baths, bulk laxatives, stool softeners, and local analgesia may be helpful. Some local anesthetics carry the risk of sensitization. Counseling to avoid precipitating factors such as prolonged standing/sitting, constipation, and delay of defecation is also appropriate.

The mother of a 6-month-old male tells you that he sometimes wheezes while feeding, and this is occasionally associated with a cough. Changing his position does not help.Which one of the following is the most likely diagnosis? (check one) A. Tracheoesophageal fistula B. Laryngeal cleft C. Gastroesophageal reflux disease D. Foreign body aspiration E. Tracheomalacia

C. Gastroesophageal reflux disease There are many causes of wheezing in infants and children. Wheezing associated with feeding is most commonly due to gastroesophageal reflux disease (level of evidence 3). Tracheoesophageal fistula and laryngeal cleft also cause wheezing associated with feeding, but are rare. Foreign body aspiration is most common between 8 months and 4 years of age and the child is most likely to have a history of the sudden onset of wheezing associated with choking. The wheezing present with tracheomalacia is position related.

A 44-year-old male in the intensive-care unit develops acute respiratory distress syndrome (ARDS). Which one of the following has been shown to improve outcomes in this situation? (check one) A. Surfactant B. Lower positive end-expiratory pressure (PEEP) settings C. Lower tidal volumes D. Aggressive fluid therapy E. Pulmonary artery catheters

C. Lower tidal volumes Acute respiratory distress syndrome (ARDS) may be caused by pulmonary sepsis or sepsis from another source, or it may be due to acute pulmonary injury, including inhalation of smoke or other toxins. Inflammatory mediators are released in response to the pulmonary infection or injury. The syndrome has an acute onset and is manifested by rapidly developing profound hypoxia with bilateral pulmonary infiltrates. The mortality rate in patients with ARDS may be as high as 55%.Early recognition and prompt treatment with intubation and mechanical ventilation is necessary to improve chances for survival. Patients with ARDS should be started at lower tidal volumes (6 mL/kg) instead of the traditional volumes (10-15 mL/kg) (SOR A). These patients also often require higher positive end-expiratory pressure settings (SOR B).Fluid management should be conservative to allow for optimal cardiorespiratory and renal function and to avoid fluid overload. However, the routine use of central venous or pulmonary artery pressure catheters is not recommended due to the potential complications associated with their use (SOR A). While surfactant is commonly used in children with ARDS, it does not improve mortality in adults (SOR A).

A 57-year-old previously healthy menopausal female presents to your office with a 1-year history of palpitations and an unintentional 10-lb weight loss. A review of systems is negative for tremors or visual changes. Vital signs include a blood pressure of 129/85 mm Hg and a heart rate of 110 beats/min. A physical examination is otherwise unremarkable except for a nontender, diffusely enlarged thyroid with no distinct nodules, and mild proptosis.Laboratory studies are significant for a TSH level<0.01 :U/mL (N 0.60-3.30), a free T3 level of 14.51 pg/mL (N 2.0-3.5), and a free T4 level of 4.52 ng/dL (N 0.71-1.40). A thyroid-stimulating immunoglobulin test is positive.In addition to a β-blocker, which one of the following is the most appropriate initial management? (check one) A. Radioactive iodine ablation B. Thyroidectomy C. Methimazole (Tapazole) D. Propylthiouracil (PTU)

C. Methimazole (Tapazole) Methimazole and propylthiouracil (PTU) are the two oral antithyroid medications available. However, because of reports of severe hepatocellular damage, methimazole should be used instead of PTU unless it is contraindicated. Radioactive iodine treatment (131I) is an option, especially for patients who do not achieve remission with antithyroid medications. However, worsening of preexisting orbitopathy is a well-recognized potential complication of 131I treatment, as well as a transient increase in thyroid hormone levels that can precipitate thyroid storm. Thus, patients with elevated free T3 or free T4 levels should be treated with methimazole prior to 131I administration. Thyroidectomy is most often recommended for patients with thyroid nodules and those who are suspected of having cancer or who do not tolerate or refuse alternative forms of therapy. However, antithyroid medication should be given to achieve a euthyroid state prior to surgery in most patients.

An 11-year-old female who plays in a local youth soccer league presents with right heel pain that has persisted for several months. She does not recall a specific injury that could have caused the pain. On examination the skin is intact and she is tender over the right posterior heel. Examination of the ankle and forefoot is unremarkable and a neurovascular examination is normal. You decide that rest and physical therapy would be the best initial management.Which one of the following is CONTRAINDICATED for this patient? (check one) A. Ice packs B. Moist heat C. Whirlpool therapy D. An exercise prescription E. Therapeutic ultrasound

Calcaneal apophysitis, also called Sever's disease, is a common cause of heel pain in young athletes, especially those who participate in basketball, soccer, track, and other sports that involve running. Typically the heel apophysis closes by age 15. Treatment options include activity modification, the use of ice packs and/or moist heat, stretching, analgesics, and orthotic devices. The use of therapeutic ultrasound on the active bone growth plates in children is contraindicated.

A 59-year-old male college professor presents with a 2-month history of right medial knee pain. There is no history of injury or overuse. He has no other specific joint pain except for occasional myalgias and arthralgias of his legs and arms. On most days he has morning stiffness lasting 15-20 minutes after getting out of bed. A review of systems is otherwise negative.On examination the right knee has full range of motion. There is tenderness at the medial joint line, but no clicking or ligamentous instability. There is crepitus with movement in both knees.Which one of the following diagnostic tests would be most appropriate at this time? (check one) A. Serologic testing B. Synovial fluid analysis C. Plain radiography D. MRI without contrast E. MRI with contrast

C. Plain radiography The most likely diagnosis for this patient's knee pain is osteoarthritis. While he is likely to have disease in both knees, it is common for patients to have unilateral symptoms, especially early on. Although osteoarthritis is mainly a clinical diagnosis, plain radiography is the diagnostic study of choice if there is concern about other diagnostic possibilities. Narrowing of the medial compartment of the knee joint is typically the first radiographic finding; osteophytes are also commonly seen on plain films. In the scenario presented here, there is no need for laboratory testing at this time. OA is based entirely on wear and tear of individual joints. RA causes prolonged morning stiffness lasting greater than 30 minutes. OA patients may feel morning stiffness, but it generally subsides within the first 30 minutes

A 30-year-old male presents to your office with a 3-week history of nausea, weight loss, diarrhea, and hematochezia. He states that he has had similar episodes twice in the past and was treated at the local urgent care clinic for infectious diarrhea, with resolution of his symptoms. Your initial laboratory workup is negative for enteric pathogens and you refer the patient for colonoscopy and esophagogastroduodenoscopy with small bowel follow-through. The patient is found to have multiple noncontiguous transmural ulcerations throughout both the small and large intestines.Which one of the following initial management strategies is most likely to induce remission in this patient? (check one) A. Laparotomy with colectomy B. Metronidazole (Flagyl) C. Prednisone D. Infliximab (Remicade)

C. Prednisone Inflammatory bowel disease is divided into two categories: Crohn's disease and ulcerative colitis. Noncontiguous or "skip" lesions that are transmural in nature and are found throughout the gastrointestinal tract make a diagnosis of Crohn's disease likely in this patient. Corticosteroids are more effective in inducing remission than placebo and 5-ASA products (SOR A). A Cochrane review revealed no difference between elemental and nonelemental diets with regard to symptom remission (SOR A). Anti-TNF agents such as infliximab should be considered in patients with moderate to severe Crohn's disease who do not respond to initial corticosteroid or immunosuppressive therapy, but these are not recommended for initial treatment. While antibiotics such as metronidazole are widely used for both their anti-inflammatory and anti-infectious properties, controlled trials have not demonstrated their effectiveness. Surgical intervention should be considered in patients with ulcerative colitis, but surgery is not indicated for Crohn's disease.

A 72-year-old male is admitted to the hospital after a syncopal episode that led to a skull fracture. All of his blood tests are in the normal range. The following morning his sodium level is 132 mEq/L (N 135-145) and further testing confirms that he is suffering from the syndrome of inappropriate secretion of antidiuretic hormone (SIADH). He is placed on a free-water restriction of <1 L/day. Later that evening he complains of a headache and vomits repeatedly. A recheck of his electrolytes shows that his sodium has dropped to 121 mEq/L.What would be the most appropriate way to address his hyponatremia at this time? (check one) A. Start oral tolvaptan (Samsca) B. Start oral sodium tablets C. Start an intravenous infusion of hypertonic saline D. Further restrict fluid intake

C. Start an intravenous infusion of hypertonic saline Head trauma is a known cause of the syndrome of inappropriate secretion of antidiuretic hormone (SIADH). This patient's course has been very acute, with hyponatremia developing within 48 hours. Such a precipitous drop in serum sodium may lead to cerebral and pulmonary edema. If left untreated the patient can have seizures, become obtunded, and die from brain herniation. These dangers require immediate treatment with hypertonic saline to correct the falling levels of sodium. This must be done cautiously so as to not overcorrect the sodium level too quickly, which could lead to osmotic demyelination syndrome. An increase in serum sodium levels of about 6 mEq/L should be enough to reduce symptoms and prevent progressive cerebral edema.

An 82-year-old white male has a cardiopulmonary arrest while mowing his lawn and his heart rhythm is restored after 8 minutes of CPR by a neighbor. He is now your patient in the coronary care unit. He is on a ventilator and has severe hypoxic encephalopathy. Echocardiography shows an ejection fraction of 12% as a result of the massive anterior myocardial infarction he sustained. Your neurology consultant confirms that the patient will never again be able to meaningfully communicate, and will be ventilator-dependent.Prior to this, the patient had been living independently and had no health problems. He has no living relatives, and his attorney confirms that he has no written advance directives. The neighbor, who is a close friend, tells you that on several occasions recently he and the patient had discussed such a scenario, and that the patient had said that if he had little chance of a meaningful recovery he would not want to remain on life support.In consultation with the hospital ethics committee, which one of the following would be most appropriate in this case? (check one) A. Transfer care of the patient to another physician B. Ask a court to appoint a guardian to make medical decisions C. Withdraw life support D. Defer the decision regarding life support to the hospital attorney E. Ask the patient's attorney to decide whether to terminate life support

C. Withdraw life support The most common form of advance directive is a patient's conversations with relatives and friends, and these carry the same ethical and legal weight as written directives. Neither the hospital attorney nor the patient's personal attorney, in the absence of a previous discussion with the patient, has the ethical or legal authority to make the decision. Since the patient has previously expressed his wishes, it is unnecessary to have the court appoint a surrogate decision maker. Care should be transferred to another physician only if the original physician has a philosophical or religious objection to carrying out the patient's wishes.

A 30-year-old female at 36 weeks gestation has a positive culture for group B Streptococcus. Her past medical history is significant for the development of a nonurticarial rash in response to penicillin.Which one of the following is most appropriate for intrapartum antibiotic prophylaxis in this patient? (check one) A. Azithromycin (Zithromax) B. Clindamycin (Cleocin) C. Vancomycin (Vancocin) D. Ampicillin E. Cefazolin

Cefazolin is appropriate for intrapartum prophylaxis against group B Streptococcus (GBS) in penicillin-allergic patients who do not have a history of anaphylaxis, urticaria, angioedema, or respiratory distress. Depending on the antibiotic sensitivity of the GBS organism, either vancomycin or clindamycin is recommended for patients at higher risk for anaphylaxis.

According to the American Diabetes Association, screening should be considered for which one of the following conditions in children with type 1 diabetes mellitus? (check one) A. Hypothyroidism B. Cystic fibrosis C. Cushing syndrome D. Systemic lupus erythematosus E. Pancreatic pseudocysts

Children with diabetes mellitus are at increased risk for retinopathy, nephropathy, and hypertension. They are also more likely to have immune-mediated disorders such as celiac disease and hypothyroidism. For all children and adolescents with type 1 diabetes mellitus, the American Diabetes Association recommends screening for hypothyroidism, nephropathy, hypertension, celiac disease, and retinopathy. Screening for dyslipidemia should be considered if there is a family history of hypercholesterolemia or cardiac events before age 55.

A 64-year-old male comes to your office for evaluation of a persistent rash affecting his groin. It is itchy but not painful and does not affect his daily activities. He has tried over-the-counter antifungal creams without relief. On examination you find well-demarcated, dark red patches in the inguinal region bilaterally. When examined with a Wood's light the area fluoresces coral-red.The most effective treatment for this condition is topical? (check one) A. Ketoconazole (Nizoral) B. Erythromycin C. Hydrocortisone D. Mupirocin (Bactroban) E. Terbinafine (Lamisil)

Coral-red fluorescence on Wood's light examination is typical of infection with Corynebacterium minutissimum, a condition known as erythrasma. This organism commonly complicates intertrigo, often in the groin or interdigital spaces. Erythromycin is the most effective treatment for this bacterial infection.

A 2½-year-old male is brought to the emergency department with the acute onset of diffuse abdominal pain that began approximately 6 hours ago. He has also had 3 episodes of bilious emesis in the last 2 hours. A review of systems is positive for anorexia today but negative for fever, weight loss, diarrhea, and bloody stools.On examination the patient's height and weight are in the 50th percentile for age, his blood pressure is normal, his heart rate is 110 beats/min, and his temperature is 36.9°C (98.4°F). Cardiovascular and pulmonary examinations are unremarkable. The abdominal examination is significant for slightly hypoactive bowel sounds and diffuse tenderness to palpation without rebound, guarding, or rigidity. A genitourinary examination is normal.Which one of the following studies is the most appropriate next step to diagnose the cause of abdominal pain in this patient? (check one) A. Scrotal ultrasonography B. Abdominal ultrasonography C. Abdominal and pelvic CT D. An upper gastrointestinal series

D. An upper gastrointestinal series In young children with bilious emesis, anorexia, and lack of fever, the most likely diagnosis is intestinal malrotation with volvulus. Abdominal ultrasonography is less sensitive and specific for malrotation than an upper gastrointestinal series, so an upper GI series should be ordered initially if volvulus is suspected. If appendicitis were suspected, ultrasonography would be preferred. CT is not a good choice because of the amount of radiation it delivers, especially given efforts to decrease the use of CT in children unless absolutely necessary. This patient's presentation is not typical for testicular torsion, therefore scrotal ultrasonography should not be the initial test of choice.

A mother brings her 10-year-old son to your office because he has recently experienced a flare-up of atopic dermatitis, including increased pruritus. Physical findings include increased erythema of the involved skin on the flexural surfaces of his arms and legs, with weeping eruptions located within areas of lichenification. Which one of the following topical treatments for managing this episode is supported by the best available evidence? (check one) A. Emollients B. Pimecrolimus (Elidel) C. Mupirocin (Bactroban) D. Corticosteroids E. Antihistamines

D. Corticosteroids Emollients are a mainstay of chronic therapy for atopic dermatitis (SOR C), but topical corticosteroids are the first-line treatment for flare-ups (SOR A). Calcineurin inhibitors such as pimecrolimus are a second-line treatment for moderate to severe atopic dermatitis (SOR A). Antibiotics are not useful in reducing flare-ups of atopic dermatitis unless there is clear evidence of a secondary infection (SOR A). Neither topical nor oral antihistamines are recommended for routine treatment of atopic dermatitis because they are not effective in treating the associated pruritus.

A 62-year-old male with a 20-year history of diabetes mellitus presents with bilateral calf and buttock pain that occurs after he walks 2 blocks. The symptoms are relieved with rest. On examination his pedal pulses are not palpable and his ankle-brachial index is 1.45.Which one of the following would be most appropriate? (check one) A. Reassuring the patient that his ankle-brachial index is normal B. MRI of the lumbar spine C. A repeat evaluation in 6 months if the symptoms persist D. MR or CT angiography of the lower extremities

D. MR or CT angiography of the lower extremities The National Health and Nutrition Examination Survey (NHANES) found that 1.4% of adults over 40 have an ankle-brachial index (ABI) >1.4; this group accounts for approximately 20% of all adults with peripheral artery disease. An ABI >1.4 indicates noncompressible arteries (calcified vessels). In patients with arterial calcification, such as diabetic patients, more reliable information is often obtained by using toe pressures to calculate a toe-brachial index and from pulse volume recordings.Vascular imaging should be used to confirm peripheral vascular disease. MR or CT arteriography, duplex scanning, and hemodynamic localization are noninvasive methods for lesion localization and may be helpful when symptoms or findings do not correlate with the ABI. Contrast arteriography is used for definitive localization before intervention.

A 45-year-old female with no significant past medical history presents to your office with 2 weeks of worsening pain in her right arm. For the past 2 months she has worked on a plastics manufacturing assembly line. A physical examination reveals no swelling and a normal range of motion. She has normal strength in the upper extremity but she experiences increased pain with extension of her right wrist against resistance. Palpation reveals marked tenderness over the lateral epicondyle of the right arm.Which one of the following is most likely to improve the patient's long-term outcome? (check one) A. Physical therapy B. Regular physical activity using her hands and arms C. Use of an inelastic, nonarticular proximal forearm strap D. Modifying her work routines

D. Modifying her work routines This patient has signs and symptoms of lateral epicondylitis, also known as tennis elbow, or alternatively as lateral epicondylalgia to reflect the noninflammatory nature of the condition. This is an overuse tendinopathy of the common extensor tendon origin of the lateral elbow. Conservative care that includes offloading the involved tendons is the key to improving outcomes at 1 year, which would mean modifying this patient's work. Physical therapy can improve pain and function in the short term, but has not been shown to improve long-term outcomes at 1 year in randomized trials (SOR A). The evidence is weaker for bracing, with some studies showing improved pain and function at 3-6 weeks (SOR B). Recent randomized, controlled trials have made it clear that while corticosteroid injections reduce acute pain for up to 6 weeks, their use increases rates of poor long-term outcomes (SOR A).

An 82-year-old female with terminal breast cancer has been admitted to hospice care. She is having severe pain that you will manage with opioids.Which one of the following would be appropriate to recommend for preventing constipation? (check one) A. Fiber supplements B. Docusate (Colace C. Metoclopramide (Reglan) D. Polyethylene glycol (MiraLax) E. No preventive measures, and treatment only if constipation develops

D. Polyethylene glycol (MiraLax) Constipation is a very common side effect of opioids that does not resolve with time, unlike many other adverse effects. Constipation is easier to prevent than to treat, so it is important to start an appropriate bowel regimen with the initiation of opioid therapy. Fiber supplements and detergents (such as docusate) are inadequate for the prevention of opioid-induced constipation. Metoclopramide is used for nausea and increases gastric motility, but is not indicated in the treatment of constipation. Polyethylene glycol, lactulose, magnesium hydroxide, and senna with docusate are all appropriate in this situation.

A 35-year-old white female complains of severe pain in her right shoulder. She notes that at night the pain intensifies if she rolls onto her right side. She has marked pain with no weakness on abduction of the shoulder; range of motion of the shoulder is normal.The most likely cause of her pain is? (check one) A. Bicipital tendinitis B. Complete rotator cuff tear C. Osteoarthritis of the shoulder D. Subacromial bursitis E. Frozen shoulder

D. Subacromial bursitis Acute subacromial bursitis is common and is often associated with calcific deposits in the supraspinatus tendon, pain on abduction, and local tenderness. Bicipital tendinitis results in tenderness on palpation of the tendon of the long head of the biceps. A rotator cuff tear usually results from an injury, and affects range of motion. Osteoarthritis seldom causes acute, severe pain. A frozen shoulder may result from subacromial bursitis and presents with limitation of shoulder motion.

A 62-year-old female with known systolic heart failure has a 2-month history of increased fatigue and worsening shortness of breath with ambulation. She says she has adhered to her medication regimen. Her oxygen saturation is 96% on room air and a physical examination is within normal limits. Laboratory studies, chest radiographs, and an EKG are ordered. The echocardiogram shows an ejection fraction of 35% and normal right heart function. Her estimated pulmonary pressure is 45 mm Hg.The best option for treatment of her pulmonary hypertension at this point is to (check one) A. add a vasodilator B. begin oxygen therapy C. recommend lifelong anticoagulation D. maximize treatment for heart failure E. schedule right heart catheterization

D. maximize treatment for heart failure This patient has pulmonary hypertension due to left heart failure. The recommended treatment is to maximize treatment for her heart failure and any other comorbidities. Vasodilators are not recommended in the treatment of pulmonary hypertension due to left heart failure and may be harmful (SOR C). Oxygen therapy is recommended only for patients with hypoxia (SOR C). Lifelong anticoagulation is recommended if pulmonary hypertension is due to chronic thromboembolic disease but not if it is due to left heart failure (SOR C). Anticoagulation is not recommended in systolic left heart failure unless there is another indication.Right heart catheterization is not recommended for pulmonary hypertension due to left heart disease because vasodilators are not a treatment option. Right heart catheterization is recommended in pulmonary hypertension prior to initiating vasodilator therapy in appropriate patients (SOR C).

A 24-year-od nulligravida comes to your office for contraception counseling. She has a seizure disorder that is well controlled on carbamazepine (Tegretol). She is a nonsmoker and has no other medical problems or complaints. She is currently in a relationship and does not want to get pregnant in the next several years.Which one of the following contraceptive options would be the most appropriate? (check one) A. Progestin-only pills B. Combined oral contraceptives C. The etonogestrel/ethinyl estradiol vaginal ring (NuvaRing D. The norelgestromin/ethinyl estradiol contraceptive patch (Ortho Evra) E. A levonorgestrel intrauterine device (Mirena)

E. A levonorgestrel intrauterine device (Mirena) Certain antiepileptic drugs induce hepatic metabolism of estrogen and progestin (carbamazepine, oxcarbazepine, phenobarbital, phenytoin, and topiramate). This can potentially lead to failure of any contraceptive that contains estrogen and progestin. Progestin-only pills are most effective in women who are exclusively breastfeeding. They are not as effective in pregnancy prevention in other circumstances. Another effective option for women taking antiepileptic medications would be an intrauterine device. The levonorgestrel (progestin only) IUD and copper IUD are acceptable choices even for a nulligravida. The single-rod implantable progestin system also would be an acceptable choice for this patient.

A 39-year-old male with a BMI of 41 kg/m2 is interested in weight loss. His medical history includes adequately controlled type 2 diabetes mellitus, well-controlled hypertension, hyperlipidemia, and obstructive sleep apnea. He has no history of coronary artery disease or COPD.Which one of the following is likely to be most effective for long-term weight loss in this patient? (check one) A. A very low calorie diet B. Increased physical activity C. Frequent, long-term weight-loss counseling D. Pharmacotherapy E. Bariatric surgery

E. Bariatric surgery Obesity increases the risk of a variety of medical conditions, including type 2 diabetes mellitus, hypertension, hyperlipidemia, pulmonary disease, coronary artery disease, gallstones, fatty liver disease, obstructive sleep apnea, GERD, osteoarthritis, and a variety of forms of cancer. A weight loss of at least 10% for greater than 1 year leads to statistically significant improvement in lipid ratios, blood glucose homeostasis, and coronary artery disease risk reduction. The AAFP recommends screening for obesity and intensive counseling (more than 1 session per month for more than 3 months) with behavior modification for obese patients. Counseling is ineffective by itself and must be combined with lifestyle modification. Dietary modification, increased physical activity, and behavior modification are effective for maintaining modest weight loss for greater than 1 year (SOR B). However, there are few large, randomized, controlled trials with subjects maintaining weight reductions of 10% for over 1 year, even when combining therapy, exercise, and dietary restriction. Long-term pharmacotherapy can lead to weight loss, but regaining some weight is typical. Bariatric surgery leads to the most effective weight reduction and long-term maintenance in patients who are morbidly obese (SOR A). Gastric bypass is effective, with a mean weight loss of 71.2% at 3 years; with laparoscopic gastric banding the mean weight loss is 55.2% at 3 years. In one study, 94% of gastric bypass patients maintained at least a 20% weight loss at 6 years. Bariatric surgery has also been shown to significantly reduce fasting blood glucose, with resolution of diabetes mellitus in 31%-77% of lap band patients and 72%-100% of gastric bypass patients. Bariatric surgery is a safe and effective means for long-term weight loss and should be considered in adults with a BMI >40, or >35 with obesity-related comorbidities.

You are counseling a 45-year-old obese male regarding weight loss. The patient has elevated triglycerides, low HDL-cholesterol, and stage 1 hypertension. He does not currently take medications and would like to avoid taking medications in the future. The patient has heard good things about low-carbohydrate diets and asks your opinion.A low-carbohydrate diet in a patient such as this is most likely to result in? (check one) A. Increased LDL-cholesterol B. Increased triglycerides C. Increased blood pressure D. Development of metabolic syndrome E. Better short-term weight loss than with a low-fat diet

E. Better short-term weight loss than with a low-fat diet Emerging data on low-carbohydrate diets is mostly encouraging, in that these diets do not seem to cause the expected increases in blood pressure, LDL-cholesterol levels, or triglyceride levels that the medical community had first assumed. Although low-carbohydrate diets have been shown to result in clinically meaningful weight loss, reduced-calorie diets appear to result in similar weight loss regardless of which macronutrients they emphasize. This patient has symptoms of metabolic syndrome and has a higher risk of glucose intolerance or diabetes mellitus. Low-carbohydrate diets have been shown to reduce insulin resistance at least as well as, if not better than, traditional diet plans.

A 35-year-old primigravid schoolteacher awakens with a rash clinically consistent with varicella early in the 38th week of her pregnancy. She had a negative varicella titer early in her pregnancy. The clinical course is mild and all vesicles have either crusted over or healed 1 week later. She has an uncomplicated labor and vaginal delivery at 40 weeks gestation, and delivers a healthy-appearing male.Of the following options, which one is the most appropriate initial management for the newborn? (check one) A. Intravenously administered varicella immune globulin B. A weight-appropriate dose of intravenous acyclovir (Zovirax) C. Varicella vaccine D. Combination treatment with varicella vaccine, intravenous acyclovir, and varicella immune globulin E. Close observation only

E. Close observation only The result of neonatal varicella infection can be catastrophic, with a fatality rate approaching 30%. Maternal immunity is ideal, but since varicella vaccination is contraindicated during pregnancy the best alternative is advising the patient to avoid contact with infected individuals until safe postpartum immunization is possible. Maternal varicella infection is particularly problematic during weeks 13-20 of pregnancy (resulting in a 2% risk of congenital varicella in the newborn) and when the onset of maternal symptoms occurs from 5 days before until 2 days after delivery.Administration of varicella immune globulin to the expectant mother has not been shown to benefit the fetus or infant, but because pregnancy can increase the risk of serious complications in the mother the Advisory Committee on Immunization Practices (ACIP) recommends that administration to pregnant women be considered following known exposure. The ACIP also recommends that term infants born within the 7-day window described above, as well as all preterm infants, receive varicella immune globulin, and that those who develop any signs of varicella infection also be given intravenous acyclovir. Term infants delivered more than 5 days after the onset of maternal varicella are thought to have adequate passive immunity for protection and the expected benign course generally requires only observation.

A 57-year-old male comes to the emergency department after several episodes of vomiting preceded by moderately severe epigastric pain. He says the vomitus looked like coffee grounds. He tells you he has had "heartburn" in the past that was sometimes severe, and occasionally associated with vomiting, but these episodes were almost always relieved by oral antacids. This problem was exacerbated recently after he began taking ketorolac for moderate arthritic pain in his knees and hands. His past medical history and a review of systems reveal no major comorbid disorders.The patient's blood pressure is 125/82 mm Hg and his heart rate is 95 beats/min with no signs of shock. His hemoglobin level is 9.5 g/dL (N 13.0-18.0). He is admitted to the hospital and placed on a proton pump inhibitor (PPI) infusion. Upper gastrointestinal endoscopy performed within 3 hours of admission shows no blood in the upper gastrointestinal tract, but reveals a Mallory-Weiss tear and a stomach ulcer containing a dark spot in an otherwise clear base.Management at this time should include which one of the following? (check one) A. Transfusion with whole blood B. Repeat endoscopy within 24 hours C. Arteriography D. Continued in-hospital observation for at least 72 hours E. Discharge from the hospital on oral PPI therapy

E. Discharge from the hospital on oral PPI therapy Blood transfusions should be administered to patients with upper gastrointestinal bleeding who have a hemoglobin level ≤7.0 g/dL (SOR C). According to the Rockall risk scoring system, this patient's mortality risk from gastrointestinal bleeding is low, based on the following: age <60, systolic blood pressure ≥100 mm Hg, heart rate <100 beats/min, no shock, and no major comorbidities. The Mallory-Weiss tear adds no points to his total score, and the only stigmata of recent hemorrhage is a dark spot in an otherwise clean ulcer base, which also adds no points. His only scored finding is the presence of the ulcer, which adds a single point to his score.Patients with low-risk peptic ulcer bleeding based on clinical and endoscopic criteria can be discharged from the hospital on the same day as endoscopy (SOR C). Routine second-look endoscopy is not recommended in patients with upper gastrointestinal bleeding who are not considered to be at high risk for rebleeding (SOR C). Arteriography with embolization is indicated only in patients with persistent bleeding. upper GI bleed -coffee grounds in vomit

A 30-year-old female presents to your office with a clear nasal discharge, sneezing, nasal congestion, and nasal itching. She notes that these symptoms generally occur in the spring and fall.The most effective drug for treatment and prevention is? (check one) A. Cetirizine (Zyrtec) B. Cromolyn nasal spray (NasalCrom) C. Ipratropium nasal spray (Atrovent D. Montelukast (Singulair) E. Fluticasone nasal spray (Flonase)

E. Fluticasone nasal spray (Flonase) The initial treatment of mild to moderate allergic rhinitis should be an intranasal corticosteroid alone, with the use of second-line therapies for moderate to severe disease (SOR A). The adverse effects and higher cost of intranasal antihistamines, as well as their decreased effectiveness compared with intranasal corticosteroids, limit their use as first-or second-line therapy for allergic rhinitis. Moderate to severe disease not responsive to intranasal corticosteroids should be treated with second-line therapies, including antihistamines, decongestants, cromolyn, leukotriene receptor antagonists, and nonpharmacologic therapies such as nasal irrigation.

Static stretching before running has been shown to? (check one) B. Increase endurance C. Decrease the frequency of lower limb muscle injury D. Reduce delayed-onset muscle soreness E. Have no benefit

E. Have no benefit Once considered generally beneficial to the running athlete, preparticipation static stretching has been found lacking in terms of benefit and even detrimental when subjected to scientific study. There is strong evidence that static stretching significantly slows performance in sprints up to 100 meters. Studies have failed to demonstrate that static stretching before running significantly decreases the likelihood of muscular injury of the lower limbs or results in a measurable reduction of delayed-onset muscle soreness. Limited evidence suggests that preparticipation static stretching, when performed alone, adversely affects both strength and endurance in elite athletes but has little measurable effect on amateur and casual athletes. Based on current understanding of sports performance, static stretching is of most benefit when performed during the cool-down period following exercise, which has been found to increase flexibility, and is best avoided immediately before athletic endeavors. A preparatory aerobic warm-up combined with dynamic range-of-motion exercises may be of some benefit for runners.

A 55-year-old male with a 4-year history of type 2 diabetes mellitus was noted to have microalbuminuria 6 months ago, and returns for a follow-up visit. He has been on an ACE inhibitor and his blood pressure is 140/90 mm Hg.The addition of which one of the following medications would INCREASE the likelihood that dialysis would become necessary? (check one) A. Hydrochlorothiazide B. Amlodipine (Norvasc) C. Atenolol (Tenormin) D. Clonidine (Catapres) E. Losartan (Cozaar)

E. Losartan (Cozaar) Patients with diabetes mellitus, atherosclerosis, and end-organ damage benefit from ACE inhibitors and angiotensin receptor blockers (ARBs) equally when they are used to prevent progression of diabetic nephropathy. Combining an ACE inhibitor with an ARB is not recommended, as it provides no additional benefit and leads to higher creatinine levels, along with an increased likelihood that dialysis will become necessary.

A 66-year-old female sees you for the first time. She has a history of iron deficiency anemia andchronic diarrhea associated with a diagnosis of celiac disease.This history increases her risk for which one of the following? (check one) A. Diverticulitis B. Ulcerative colitis C. Crohn's disease D. Colon cancer E. Osteoporosis

E. Osteoporosis Patients who are diagnosed with celiac disease are at increased risk of osteoporosis due to bone loss from decreased calcium and vitamin D absorption. These patients are at higher risk for fractures. Patients with celiac disease are not at increased risk for inflammatory bowel disease, diverticulitis, or colon cancer.

A 39-year-old male presents to the emergency department with a 2-hour history of chest discomfort, dyspnea, dizziness, and palpitations. He has no history of coronary artery disease. He states that he has had several similar episodes in the last year. On examination he has a temperature of 36.8°C (98.2°F), a respiratory rate of 25/min, a heart rate of 193 beats/min, a blood pressure of 134/82 mm Hg, and an O2 saturation of 96% on room air. The physical examination is otherwise normal. An EKG reveals a regular narrow QRS complex tachycardia with no visible P waves.He converts to normal sinus rhythm with intravenous adenosine (Adenocard). Which one of the following would be most useful in the long-term management of this patient's condition? (check one) A. Adenosine B. Digoxin C. Vagal maneuvers D. Pacemaker placement E. Radiofrequency ablation

E. Radiofrequency ablation This patient presents with a classic description of supraventricular tachycardia (SVT). The initial management of SVT centers around stopping the aberrant rhythm. In the hemodynamically stable patient initial measures should include vagal maneuvers (SOR C), intravenous adenosine or verapamil (SOR B), intravenous diltiazem or β-blockade, intravenous antiarrhythmics, or cardioversion in refractory cases. While digoxin is occasionally useful in atrial fibrillation with a rapid ventricular rate, it is not recommended for SVT. Radiofrequency ablation is fast becoming the first-line therapy for all patients with recurrent SVT, not just those refractory to suppressive drug therapies. Observational studies have shown that this therapy results in improved quality of life and lower cost as compared to drug therapy (SOR B).

A 25-year-old male has developed a painless ulcer on the glans of his penis. After an appropriate examination and testing you diagnose primary syphilis and treat him with 2.4 million units of benzathine penicillin intramuscularly in a single dose. Eight hours later, while you are working the evening clinic, he returns because he has a fever of 100.6°F and a bad headache, which he rarely gets. He says he "aches all over."Which one of the following would be most appropriate at this time? (check one) A. Three blood cultures from different sites at 30-minute intervals B. CT of the head C. A lumbar puncture D. Doxycycline, 100 mg orally twice a day for 14 days E. Reassurance and antipyretics

E. Reassurance and antipyretics This patient is experiencing the Jarisch-Herxheimer reaction—an acute, transient, febrile reaction that occurs within the first few hours after treatment for syphilis. The condition peaks at 6-8 hours and disappears within 12-24 hours after therapy. The temperature elevation is usually low grade, and there is often associated myalgia, headache, and malaise. It is usually of no clinical significance and may be treated with salicylates in most cases. The pathogenesis of the reaction is unclear, but it may be due to liberation of antigens from the spirochetes.

A 42-year-old female is troubled by her lack of interest in sex. She is generally healthy, takes no medications, and has regular menstrual periods. She is content with the emotional intimacy of her marriage and has had satisfying sexual interactions in the past. She does not have any religious or cultural barriers regarding her sexuality, and asks for ideas on how to improve her situation.Which one of the following has consistent evidence of benefit in cases such as this? (check one) A. Cognitive-behavioral therapy B. Viewing pornography C. Oral estrogen D. Oral sildenafil (Viagra) E. Topical testosterone

E. Topical testosterone This patient meets the criteria for hypoactive sexual desire disorder (HSDD). The incidence of this condition is variable based on the age, life stage, and culture of the patient, but is estimated to be present in about 5%-15% of the adult female population. This diagnosis includes two components: (1) recurrent deficiency or absence of sexual desire or receptivity to sexual activity, and (2) distress about such a deficiency. In menstruating women, oral estrogen and oral sildenafil have not been shown to be superior to placebo. Cognitive-behavioral therapy has been shown to be helpful for other sexual dysfunctions, but not with HSDD. Topical testosterone, in either patch or gel form, has shown consistent improvements in arousal, desire, fantasy, orgasm, and overall satisfaction in cases of HSDD.

A 70-year-old male without underlying lung disease presents with a 36-hour history of fever, body aches, cough, and dyspnea. He did not receive influenza vaccine this year, and was recently exposed to his grandson who had influenza.On examination the patient has a temperature of 38.8°C (101.8°F), a blood pressure of 90/50 mm Hg, a heart rate of 110 beats/min, and an O2 saturation of 87% on room air. A nasal swab rapid antigen test is negative, and his WBC count is 15,000/mm3 (N 4300-10,800). A viral culture is sent to the laboratory. A chest radiograph shows a large lobar pneumonia.You hospitalize the patient and initiate? (check one) A. ceftriaxone (Rocephin) and azithromycin (Zithromax) B. levofloxacin (Levaquin) C. oseltamivir (Tamiflu) D. oseltamivir, ceftriaxone, and azithromycin E. oseltamivir, ceftriaxone, azithromycin, and vancomycin (Vancocin)

E. oseltamivir, ceftriaxone, azithromycin, and vancomycin (Vancocin) This patient has pneumonia, sepsis, and suspected coinfection with influenza. Although the rapid antigen-based nasal swab was negative, false-negative rates may be as high as 70% and this test should not be relied upon to rule out influenza. Treatment should include both antiviral and antibacterial agents that include coverage against methicillin-resistant Staphylococcus aureus (MRSA), the most common bacterial pathogen isolated from critically ill patients with coinfection. Oseltamivir, ceftriaxone, azithromycin, and vancomycin should be initiated empirically for the pneumonia and sepsis. The criteria for sepsis are satisfied by a temperature >38.3°C, a WBC count >12,000/mm3, a respiratory rate >20/min, and a source of probable infection.

A 58-year-old female with diabetes mellitus complains of 2 years of right shoulder pain, which is worse with activity. There has been no trauma. She tells you one of her friends had a similar problem and was treated successfully with "some sort of shock wave treatments."Which one of the following diagnoses is most likely to be successfully treated with extracorporeal shock wave therapy? (check one) A. Calcific tendinitis B. Gout C. Partial rotator cuff tear D. Frozen shoulder E. Hooked acromion

Extracorporeal shock wave therapy is effective for calcific tendinitis of the rotator cuff. Side effects include bruising and pain. Needling and irrigation, physical therapy, and cortisone injections are sometimes used in patients with acute symptoms. Endoscopic and open surgical treatments are alternatives to extracorporeal shock wave therapy in refractory cases. Extracorporeal shock wave therapy does not have an established role in gout, rotator cuff tear, frozen shoulder, or hooked acromion.

Traveler's diarrhea can be effectively treated in the great majority of cases with which one of the following? (check one) A. Erythromycin B. Penicillin C. Sulfacetamide D. Ciprofloxacin (Cipro)

Fluoroquinolones such as ciprofloxacin have been shown to significantly reduce the duration and severity of traveler's diarrhea when given for 1-3 days. Sulfacetamide is available only in a topical form for use in the eye. Penicillin and erythromycin are not effective against the most common cause of traveler's diarrhea, enterotoxigenic Escherichia coli.

The Timed Up and Go test consists of a patient rising from a chair, walking 3 meters (or about 10 feet), turning around, walking back, and sitting back down. The average healthy adult over the age of 60 can perform this in how many seconds? (check one) A. 5 B. 10 C. 20 D. 30 E. 45

For the average adult over the age of 60, the normal time required for the Timed Up and Go test is 10 seconds. A time longer than 10 seconds may indicate weakness, a balance or gait problem, and/or an increased fall risk.

A 40-year-old male complains of a cough that has persisted for more than 3 months. He is otherwise asymptomatic. A chest radiograph and pulmonary tests are normal.Which one of the following is the most likely cause? (check one) A. Bronchiectasis B. Tuberculosis C. Sarcoidosos D. Asthma E. Gastroesophageal reflux disease

Gastroesophageal reflux disease is one of the most common causes of chronic cough. Patients with "silent" gastroesophageal reflux may not have the classic symptoms of heartburn and regurgitation. The diagnosis is based on resolution of the cough with an empiric trial of a proton pump inhibitor, although a chest radiograph should be obtained in all patients with a chronic cough to exclude bronchiectasis, tuberculosis, and sarcoidosis. Asthma is another frequent cause of chronic cough, but it can be ruled out with normal pulmonary function tests.

Which one of the following is associated with a herald patch? (check one) A. Pityriasis alba B. Pityriasis lichenoides C. Pityriasis rosea D. Pityriasis rubra pilaris E. Pityriasis (tinea) versicolor

In 50%-90% of patients, pityriasis rosea starts with an erythematous, scaly, oval patch a few centimeters in diameter. This is usually followed within a few days by smaller patches on the trunk and sometimes the proximal extremities. Pityriasis rubra pilaris is a rare disease with five types. The classic adult type begins with a small red plaque on the face or upper body that gradually spreads to become a generalized eruption. The other conditions listed typically begin with multiple lesions.

Which one of the following juices can greatly increase the blood level of a statin? (check one) A. Apple B. Grapefruit C. Orange D. Pineapple E. Tomato

Ingestion of grapefruit juice can increase absorption and serum levels of statins, leading to an increased risk of muscle injury. The mechanism for this is believed to be the cytochrome p-450 pathway. Starfruit juice and pomegranate juice can have a similar effect. These juices contain an irreversible inhibitor of intestinal CYP3A4, and increase the bioavailability of atorvastatin, lovastatin, and simvastatin. Rosuvastatin and fluvastatin utilize the CYP2C9 system for metabolism, so the effect on these drugs is minimal.Grapefruit juice reduces CYP3A4 activity by 50% within 4 hours of ingestion, and activity is reduced by 30% for as long as 24 hours after ingestion. Several studies document that consuming 600 mL of double-strength juice for 3 days produces a more than tenfold increase in the area under the curve for simvastatin and lovastatin, but only a 250% increase in atorvastatin.

For which one of the following respiratory infections should antibiotic therapy be initiated immediately upon diagnosis? (check one) A. Bronchitis B. Epiglottitis C. Laryngitis D. Rhinosinusitis E. Tracheitis

Many infections of the respiratory tract have a viral etiology, and when this is the case early antibiotic treatment offers little to no benefit. Once the clinical course of a respiratory illness exceeds the expected length for a viral illness, it may be proper to initiate antibiotic treatment for a suspected atypical or secondary bacterial infection. Epiglottitis is one exception to this approach because of the possibility of a bacterial infection, particularly with Haemophilus influenzae type b, that can produce a rapidly worsening, potentially fatal airway compromise. When epiglottitis is suspected based on findings such as hoarseness, dysphagia, stridor, drooling, fever, chills, and respiratory distress, intravenous antibiotic treatment should be instituted immediately, ideally with a $-lactam drug that exhibits activity against methicillin-resistant Staphylococcus aureus.

A 46-year-old African-American female sees you because of a history of excessive uterine bleeding and irregularity in her menstrual cycle. She has three children and had a tubal ligation after her last delivery. A pelvic examination does not reveal any pathology to explain her symptoms. Further laboratory evaluation indicates that she is mildly anemic. You perform an endometrial biopsy in the office that confirms your suspicion of endometrial hyperplasia without atypia.Which one of the following is the treatment of choice for this patient? (check one) A. Elective hysterectomy B. Hysteroscopic endometrial laser ablation C. High-dose oral estrogen supplementation D. Antifibrinolytic therapy E. Progestational drugs

Medical therapy with progestational drugs is the treatment of choice for menorrhagia due to endometrial hyperplasia without atypia. Progestins convert the proliferative endometrium to a secretory one, causing withdrawal bleeding and the regression of hyperplasia. The most commonly used form is cyclic oral medroxyprogesterone, given 14 days per month, but implanted intrauterine levonorgestrel is the most effective (SOR A) and also provides contraception. High-dose estrogen supplementation would further stimulate the endometrium. Estrogen is useful in cases where minimal estrogen stimulation is associated with breakthrough bleeding. The anti-fibrinolytic agent tranexamic acid prevents the activation of plasminogen and is given at the beginning of the cycle to decrease bleeding. Side effects and cost limit this treatment option, however. It may be most useful in women with bleeding disorders or with contraindications to hormonal therapy.NSAIDs, which decrease prostaglandin levels, reduce menstrual bleeding but not as effectively as progestins. While mefenamic acid is marketed for menstrual cramps and bleeding, all NSAIDs have a similar effect in this regard. If medical management fails, hysteroscopic endometrial ablation is an option for reducing uterine bleeding but is considered permanent and obviously will impair fertility. Hysterectomy is reserved for severe and chronic bleeding that is not relieved by other measures.

A 62-year-old female presents to your office with diarrhea and signs and symptoms ofdehydration. She has a temperature of 38.6°C (101.5°F) and a WBC count of 17,000/mm3 (N5300-10,800). You admit her to the hospital, and a Clostridium difficile toxin assay is positive.Because of the severity of her infection, you initiate oral vancomycin (Vancocin), 125 mg 4times daily. She has a poor clinical response and you decide to alter the antibiotic regimen toinclude intravenous coverage.Which one of the following intravenous antibiotics would be most appropriate? (check one) A. Ciprofloxacin (Cipro) B. Imipenem/cilastatin (Primaxin) C. Meropenem (Merrem) D. Metronidazole E. Vancomycin

Metronidazole, vancomycin, and fidaxomicin are the three medications recommended for treatment ofClostridium difficile colitis infections. Only metronidazole is effective intravenously, because its biliary excretion and possibly exudation through the colonic mucosa allows it to reach the colon via the bloodstream. Treatment for this condition with vancomycin and fidaxomicin is oral. Imipenem/cilastatin,ciprofloxacin, and meropenem have not been shown to be effective for C. difficile infection.

A 44-year-old female with localized breast cancer is receiving counseling about adjuvant long-term therapy. Which one of the following is more likely to occur with an aromatase inhibitor such as letrozole (Femara) than with tamoxifen (Soltamox)? (check one) A. Endometrial cancer B. Venous thromboembolism C. Inflammatory arthritis D. Myalgias

Myalgias and noninflammatory arthralgias are more likely with aromatase inhibitors. Venous thromboembolism rarely occurs with these drugs. Endometrial cancer may occur with long-term use of tamoxifen.

A 78-year-old asymptomatic male is found to have a platelet count of 90,000/mm3 (N 150,000-300,000) and a slightly decreased WBC count. Which one of the following would be most consistent with a diagnosis of myelodysplastic syndrome? (check one) A. A normal RBC count and indices B. Normocytic anemia C. Microcytic anemia D. Macrocytic anemia E. Polycythemia

Myelodysplastic syndrome is a hematologic malignancy with a predisposition to leukemic transformation. It can present with findings of anemia, thrombocytopenia, neutropenia, or any combination of these. Anemia occurs in 80%-85% of patients and is typically macrocytic.

Long-term alleviation of carpal tunnel syndrome in patients with persistent symptoms is best accomplished by which one of the following? (check one) A. Splinting B. Physical therapy C. Ibuprofen D. Corticosteroid injection E. Surgery

NSAIDs, pyridoxine, and diuretics have been shown to be no more effective than placebo in the treatment of patients with carpal tunnel syndrome. Splinting, physical therapy, and corticosteroid injections have all been shown to result in short-term improvement. Patients with persistent symptoms achieve the best long-term relief with surgery.

A 63-year-old male with a history of alcoholism and compensated hepatic cirrhosis asks if there are pain medications he can use to treat his chronic low back pain and knee and hand osteoarthritis. He also has occasional headaches. He has not used alcohol for several years.Which one of the following medications is CONTRAINDICATED in this patient? (check one) A. Acetaminophen B. Gabapentin (Neurontin) C. Naproxen D. Pregabalin (Lyrica) E. Tramadol (Ultram)

Naproxen Although patients with chronic mild liver disease may take NSAIDs, they should be avoided in all patients with cirrhosis, due to the risk of precipitating hepatorenal syndrome. Pregabalin and gabapentin are not metabolized by the liver and can be quite helpful. Acetaminophen, while toxic in high doses, can be used safely in dosages of 2-3 g/day. Tramadol is also safe in patients with cirrhosis

Human parvovirus B19 is associated with which one of the following? (check one) A. Erythema marginatum B. Erythema multiforme C. Erythema toxicum D. Erythema infectiosum E. Erythema chronicum

Parvovirus B19 is associated with erythema infectiosum, or fifth disease. It is also associated with nonspecific fever, arthropathy, chronic anemia, and transient aplastic crisis.

According to national and international guidelines, which one of the following is the next step for adults with asthma who require therapy with inhaled β-agonists more than three times a week? (check one) A. Inhaled glucocorticoids B. Inhaled salmeterol (Serevent) C. Sustained-release oral β-agonists D. Sustained-release oral theophylline

Patients who require inhalation therapy with β2-adrenergic-receptor agonists more than twice weekly but not daily have mild persistent asthma. Long-term control with inhaled corticosteroids is recommended for adults with persistent asthma.

Which one of the following is most suggestive of plantar fasciitis? (check one) A. Heel pain at rest B. A heel spur on radiographs C. Prompt relief with NSAIDs D. Heel pain that is worse with the first steps in the morning

Plantar fasciitis is characterized by pain that is worse with the first few steps in the morning or after a prolonged rest. NSAIDs may help with the discomfort, but prompt relief of the pain by any modality is not common. The pain is typically in the medial heel. While 50% of people with plantar fasciitis have heel spurs on radiographs, this finding is not causative or diagnostic. The diagnosis is made clinically.

An 85-year-old navy veteran presents to your office with a complaint of cough and dyspnea with exertion. He spent his entire career in ship maintenance and repair, and retired from the navy at the age of 45. His chest radiograph shows pleural plaques. He has a 20-pack-year smoking history, but quit at the age of 39.You suspect his problem is due to occupational exposure to which one of the following? (check one) A. Asbestos B. Beryllium C. Iron oxide D. Silica E. Uranium

The inhalation of asbestos fibers may lead to a number of respiratory diseases, including lung cancer, asbestosis, pleural plaques, benign pleural effusion, and malignant mesothelioma. High-risk populations for asbestos exposure include individuals who worked in construction trades or as boilermakers, shipyard workers, or railroad workers, as well as U.S. Navy veterans. The occupational history helps to guide clinical suspicion in these high-risk populations. This patient is a retired U.S. Navy veteran who spent his entire career in ship maintenance and repair.The patient history is not consistent with berylliosis, silicosis, or uranium exposure. Berylliosis is an occupational disease related to mining and manufacturing. Silicosis is seen in sandblasters, miners, persons who have worked with abrasives, and several other occupations. Uranium exposure occurs after nuclear reactor leaks or blasts. Uranium compounds are also used in photography and as dyes or fixatures. The chemical toxicity involves nonmalignant damage to alveolar cells. Iron oxide exposure is not known to be related to lung disease.

A 32-year-old gravida 2 para 1 with long-standing untreated hypertension presents at 8 weeks gestation for prenatal care. Her physical examination is normal except for a blood pressure of 156/114 mm Hg.Which one of the following would be most appropriate as initial treatment? (check one) A. Labetalol (Trandate) B. Lisinopril (Prinivil, Zestril) C. Losartan (Cozaar) D. Metoprolol (Lopressor, Toprol-XL) E. Nifedipine, immediate release (Procardia)

The drug most often recommended as first-line therapy for hypertension in pregnancy is labetalol. Reports of an association of metoprolol with fetal growth restriction have given rise to the recommendation to avoid its use in pregnancy. Both ACE inhibitors and angiotensin-receptor blockers are contraindicated in pregnancy because of the risk of birth defects and fetal or neonatal renal failure. Immediate-release nifedipine is not recommended due to the risk of hypotension.

A 70-year-old retired farmer presents with an angulated right knee and a painful hip. He asks you about the possibility of having knee replacement surgery, although he is not eager to do so.You would advise him that the major indication for knee replacement is? (check one) A. Severe joint pain B. Marked joint space narrowing on radiologic studies C. Destruction and loss of motion of the contralateral joint D. An acutely infected joint

The major indication for joint replacement is severe joint pain. Loss of joint function and radiographic evidence of severe destruction of the joint may also be considered in the decision. The appearance of the joint and the status of the contralateral joint may be minor considerations. Surgical insertion of a foreign body into an infected joint is contraindicated.

A 15-year-old male presents to the emergency department at 10 p.m. with a 2-hour history ofsevere, acute scrotal pain associated with vomiting. On examination the right testicle is swollen.Ultrasonography is inconclusive.Which one of the following would be most appropriate at this point? (check one) A. Repeat ultrasonography in the morning B. Antibiotics C. Corticosteroids D. Scrotal support E. Immediate surgical consultation

The patient has typical signs and symptoms of testicular torsion despite inconclusive ultrasonography.Surgical exploration is necessary because the testicle can be salvaged if the torsion is repaired within 6hours of symptom development (SOR C).

The primary indication for joint replacement surgery in patients with osteoarthritis is? (check one) A. intractable pain B. joint laxity C. limited range of motion D. recurrent subluxation

The primary indication for joint replacement surgery in patients with osteoarthritis is intractable pain, which is almost always relieved by the surgery. Joint replacement may also be appropriate for patients with significant limitations of joint function or with altered limb alignment. Range of motion, joint laxity, and recurrent subluxation relate to musculotendinous function, and are not reliably improved by joint replacement.

Which one of the following has good evidence of effectively improving borderline personality disorder? (check one) A. SSRIs B. Second-generation antipsychotics C. Omega-3 fatty acids D. No currently available pharmacotherapy

There are no proven therapies to reduce the severity of borderline personality disorder (SOR A). The most promising psychological therapy is dialectic behavioral therapy (DBT). DBT is a multi-faceted program specifically designed to treat borderline personality disorder. The few, small studies of DBT found improvement in many symptoms of borderline personality disorder, but long-term data is lacking. Another promising therapy is psychoanalytic-oriented day hospital therapy. Again, study sizes have been small and data cannot be extrapolated to the population as a whole.Omega-3 fatty acids, second-generation antipsychotics, and mood stabilizers have been shown to be helpful for some symptoms of borderline personality disorder but not for overall severity. Their benefits are based on single-study results and side effects were not addressed in the studies. SSRIs are not recommended for borderline personality disorder unless there is a concomita

A 20-year-old female distance runner presents with a 1-month history of left knee pain. The pain is worse with her first few steps in the morning and when going down stairs. Examination of the knee reveals no deformity, effusion, or ligamentous laxity. The knee joint and surrounding tissues are not tender to palpation, with the exception of an area 2 cm proximal to the left lateral joint line.What is the most likely cause of this patient's pain? (check one) A. Osteoarthritis of the knee joint B. Pes anserine bursitis C. Iliotibial band syndrome D. Chronic lateral meniscal tear E. Osgood-Schlatter disease

This case illustrates the classic history and physical findings of iliotibial band syndrome. Pain occurs most frequently at the site where the tendon crosses over the lateral femoral epicondyle. With osteoarthritis or a meniscal tear there would be pain in the joint space with palpation. Osgood-Schlatter disease is more common in younger adolescents and is characterized by tenderness of the tibial tubercle at the distal insertion of the patellar ligament. Pes anserine bursitis is characterized by pain in the medial knee distal to the joint space, at the conjoined tendon of the sartorius, gracilis, and semitendinosus.

A 40-year-old male presents with a sudden onset of unilateral peripheral facial nerve weakness 1 hour ago. Which one of the following is most likely to shorten his symptoms? (check one) A. Corticosteroid therapy B. Antiviral therapy C. Thrombolytic therapy D. Hyperbaric oxygen therapy E. Facial nerve decompression

This patient has Bell's palsy. Only corticosteroids have been shown to improve the outcome. Antiviral agents have little value in the treatment of Bell's palsy. Thrombolytic therapy may be useful for a patient with central facial nerve weakness if it is due to a vascular event (level of evidence 3; SOR A).

A 70-year-old white female asks you to evaluate her right shoulder because of pain and limited range of motion. Further history reveals that 2 months ago she slipped in her kitchen and caught onto the refrigerator door handle to avoid falling to the floor. On examination she has pain and weakness at 45° of abduction and weakness on external rotation.She should be treated for? (check one) A. Bicipital tendinitis B. Disruption of the glenoid fossa C. Rotator cuff tear D. Acromioclavicular separation E. Incomplete fracture of the humeral head

This patient has a history and physical findings that are consistent with a rotator cuff tear. Most commonly the mechanism of injury in an acute rotator cuff tear is forced abduction of the arm with significant resistance. Often this will occur when a person attempts to break a fall with an outstretched hand. There is usually a sudden sensation of tearing pain in the shoulder. Pain and muscle spasm will limit shoulder motion. Patients with a large tear cannot initiate shoulder abduction and will have a discrepancy between active and passive motion. Patients with significant tears will also have a positive drop arm test. This test is performed by passively abducting the arm to 90° and asking the patient to hold the arm in that position while the examiner applies pressure on the distal forearm or wrist. The test is positive if the pressure causes the arm to drop suddenly.Acute tears are generally managed with a splint and orthopedic referral for surgical repair. Chronic tears may be managed with shoulder rehabilitation but may ultimately require surgical repair as well.Bicipital tendinitis is not generally caused by acute trauma, but by irritation and microtrauma due to repetitive elevation or abduction of the shoulder, causing an inflammatory reaction in the synovial sheath. Patients generally present with a complaint of pain in the anterior shoulder that radiates into the upper arm. It is more painful with activity and is worse at night. Abduction and external rotation of the arm exacerbates the pain. On examination there should be point tenderness in the bicipital groove. Active range of motion will be limited by pain but passive range of motion will be intact. There should not be any weakness.Acromioclavicular separation is usually caused by a fall or a direct blow to the point of the shoulder with the shoulder abducted. The pain associated with this injury is over the acromioclavicular joint margin and there may be swelling. Depending on the severity of the injury there may be full range of motion but it may be restricted due to pain. There should not be any weakness associated with this injury.A fracture of the humeral head generally occurs with a fall on an outstretched arm or direct blow to the lateral side of the arm. Generally there is pain or bruising over the fracture site. Movement will be restricted by pain, but there should not be any weakness.A tear of the labrum can occur with acute trauma or from repetitive shoulder motion. Acute trauma may occur from a dislocation of the shoulder, falling on an outstretched arm, or direct blows to the shoulder. Generally, people with a tear of the labrum will have increased pain with overhead activity, popping or grinding, loss of strength, and trouble locating a specific point of pain.

An 18-year-old basketball player comes to your office for evaluation of finger pain. During a basketball game yesterday he was hit on the tip of his right second digit and now has finger pain and difficulty moving his finger. On examination he has bruising and tenderness over the distal interphalangeal (DIP) joint. His DIP joint is in the flexed position and he is unable to extend the joint. A radiograph shows a fracture at the dorsal surface of the proximal distal phalanx involving 10% of the joint space.What is the most appropriate management of this injury? (check one) A. Taping the finger to the adjacent finger B. Splinting in full extension C. Splinting in 45° of flexion D. Urgent surgical management E. Intermittent splinting for comfort

This patient has a mallet fracture. These fractures are caused by an axial load to the tip of an extended finger that causes forced flexion at the distal interphalangeal (DIP) joint. This leads to a fracture at the dorsal surface of the proximal distal phalanx where the terminal finger extensor mechanism inserts. The most appropriate treatment of a mallet fracture is to splint the DIP joint in extension for 8 weeks. The joint should remain in full extension for optimal healing. Any flexion of the finger may affect healing and extend the treatment time.Surgical management has been recommended for fractures that involve more than 30% of the joint space, but a small study showed there was no difference in outcomes compared to treatment with extension splints. Buddy taping would not offer enough support to maintain the finger in extension at all times.

A 32-year-old female has a 3-week history of depressed mood. She reports markedly diminished interest or pleasure in most activities, fatigue, a diminished ability to concentrate, and insomnia. She has had recurrent suicidal thoughts, but has no specific plan. Further investigation reveals a past history of several hypomanic episodes lasting 4-5 days, characterized by a persistently elevated, expansive mood. During these episodes she needed little sleep, was talkative, met multiple goals, and had trouble keeping up with the thoughts that were running through her head. She was treated with lithium in her early twenties but she stopped taking it because it stifled her artistic creativity. She currently takes no medication. Her physical examination is unremarkable. Results from comprehensive laboratory studies, including a urine toxicology screen, are also normal. Which one of the following is most appropriate for her current depressive symptoms? (check one) A. Aripiprazole (Abilify) B. Venlafaxine C. Divalproex (Depakote) D. Divalproex and bupropion (Wellbutrin) E. Lithium and paroxetine (Paxil)

This patient has bipolar II disorder. She has a history of hypomanic episodes as well as major depression, with no history of a manic or mixed episode. Among the pharmacologic options listed, only divalproex and lithium are indicated for treating bipolar depression or acute mania, and for maintenance. They should be given as single agents, however, not in combination with other drugs. No evidence supports combination therapy or the addition of an antidepressant in the acute phase of depression.In a study of patients with bipolar II disorder, initially adding paroxetine or bupropion to the mood stabilizer was no more effective than using lithium or valproate. An SSRI or bupropion can be added if a therapeutic dosage of a mood stabilizer does not resolve symptoms and the patient is not in a mixed state. Tricyclic antidepressants and antidepressants with dual properties, such as venlafaxine, should be avoided because they may induce mania. Aripiprazole is indicated for acute mania but not for bipolar depression.

A 54-year-old female presents with painful sores in her mouth that appeared a few days ago. She has had some trouble eating due to the pain, but she is able to swallow without difficulty. She also began to have some pain around her right ear today. She has no fever, chills, nasal congestion, cough, or difficulty hearing. Her medical history is significant only for an anxiety disorder treated with sertraline (Zoloft). On examination her vital signs are all normal. You see vesicles on the right side of the hard palate and she has a swollen, red right pinna, with vesicles in the external auditory canal.The organism responsible for this condition is (check one) A. coxsackievirus B. Epstein-Barr virus C. group A Streptococcus D. herpes simplex virus E. varicella zoster virus

This patient has herpes zoster oticus, which is also known as Ramsay Hunt syndrome when associated with a facial nerve palsy. It is caused by reactivation of the varicella-zoster virus (VZV) in the geniculate ganglion of the facial nerve. Typical symptoms include painful vesicles on one side of the palate and the ipsilateral ear. When the reactivation involves other branches of the facial nerve it can result in a unilateral facial herpetiform rash that may also involve the anterior two-thirds of the tongue, taste disturbance, and reduced lacrimation. If the nearby cochlear and vestibular nerves become involved, patients may also experience hearing loss, tinnitus, nausea, vomiting, and vertigo. The diagnosis is usually made clinically, but if confirmation is needed polymerase chain reaction testing of vesicular fluid or of a swab of the base of an ulcer may be done. Treatment includes antivirals (acyclovir, valacyclovir) and prednisone, and is more effective when started sooner in the course of illness.Herpes simplex virus (HSV) can cause oral vesicles and ulcers, but the distribution of vesicles in the ear and the mouth of this patient is not typical for HSV. Epstein-Barr virus can cause leukoplakia of the mouth but not vesicles and is typically associated with systemic signs of illness. Group A Streptococcus causes throat pain and fever, not vesicles. Coxsackievirus causes oral vesicles and ulcers but is usually associated with fever and does not typically involve the ear.

An 80-year-old male nonsmoker with Parkinson's disease is treated for community-acquired pneumonia with azithromycin (Zithromax), 500 mg/day for 10 days. On follow-up the patient feels better but still has a productive cough. A repeat chest radiograph reveals a single thin-walled cavity lesion in the left lower lobe.It would be most appropriate to replace this patient's azithromycin with? (check one) A. Doxycycline B. Clindamycin (Cleocin) C. Metronidazole (Flagyl) D. Trimethoprim/sulfamethoxazole (Bactrim, Septra)

This patient most likely has an anaerobic bacterial infection. Penicillin was used to treat these infections in the past, but because of the emergence of β-lactamase-producing organisms, clindamycin is now the drug of choice. Clindamycin has broader coverage against both pulmonary anaerobes and facultative aerobes such as Staphylococcus aureus and Klebsiella, which are often seen with lung abscesses. Metronidazole has anaerobic coverage, but not for the anaerobic species often involved in pulmonary infections, and is therefore associated with a high failure rate when used to treat lung abscesses. Doxycycline does not cover anaerobes. Trimethoprim/sulfamethoxazole is also not considered a good anaerobic antibiotic.

A morbidly obese 68-year-old male complains of breast enlargement. He has not noticed any pain or discomfort from this problem. His past medical history is negative except for type 2 diabetes mellitus and hypertension. His medications include metformin (Glucophage), 1000 mg twice daily; lisinopril (Prinivil, Zestril), 20 mg daily; and aspirin, 81 mg daily. His family history is negative for breast cancer. A physical examination is negative except for a BMI of 45 kg/m2 and symmetric bilateral adipose tissue in the breast region on inspection and palpation. There is no glandular tissue on careful palpation of the area beneath the areolae and nipples. No nodules or axillary nodes are detected. There is no nipple retraction or discharge, and no skin changes.Which one of the following is the most likely cause of this problem? (check one) A. Fat necrosis B. Gynecomastia C. Pseudogynecomastia D. Breast cancer E. Mastitis

This patient most likely has pseudogynecomastia due to increases in subareolar fat secondary to his obesity. This is based upon clinical findings of symmetric adipose tissue in the breast region bilaterally and a lack of firm, palpable glandular tissue in the nipple and areolar region. In gynecomastia, there is palpable, firm glandular tissue in a concentric mass around the nipple-areola complex. Hard, immobile masses, masses associated with skin changes, nipple retraction, nipple discharge, or enlarged lymph nodes would suggest possible malignancy. Fat necrosis would involve a history of breast region trauma and would generally be asymmetric. Mastitis would cause clinical signs of infection.

72-year-old female who remains very active and engaged in the community comes to youroffice concerned by urinary symptoms that disrupt her life. She reports that she often has astrong, abrupt desire to void that frequently causes her to leak urine involuntarily. She alsoreports occasional episodes of urinary frequency and nocturia.Which one of the following is the first-line treatment for her condition? (check one) A. Anticholinergic drugs such as oxybutynin or solifenacin (Vesicare) B. β-Adrenergic agonists such as mirabegron (Myrbetriq) C. Duloxetine (Cymbalta) D. Bladder training E. A pessary

This patient suffers from urge urinary incontinence, defined as the loss of urine accompanied or precededby a strong impulse to void. It may be accompanied by frequency and nocturia, and is common in olderadults. Conservative therapies such as behavioral therapy, including bladder training and lifestylemodification, should be the first-line treatment for both stress and urge urinary incontinence (SOR C).Pharmacologic interventions should be used as an adjunct to behavioral therapies for refractory urge incontinence (SOR C). Vaginal inserts, such as pessaries, can be used for treating stress incontinence but not urge incontinence.

A 6-month-old male is brought in for a routine checkup. Only one testicle is palpable. The genital examination is otherwise within normal limits.Which one of the following would be most appropriate at this time? (check one) A. Observation only, until 18 months of age B. Abdominal ultrasonography C. Urologic referral for surgical exploration D. HCG treatment for 3 months

Treatment for a unilateral undescended testis should be started at 6-12 months of age to avoid testicular damage. It was once thought that delaying descent lowered the incidence of testicular cancer, but it is now believed that orchiopexy allows for early cancer detection. HCG treatment may promote descent into the distal canal, but the testicle often ascends again. Ultrasonography will not show an undescended testis in many cases and is therefore not recommended. Hormonal treatments have been used in Europe but randomized, controlled trials have not shown them to be effective.

A 69-year-old female presents with her first episode of Clostridium difficile colitis, which ischaracterized as severe. Which one of the following is the most appropriate initial therapy? (check one) A. Oral metronidazole (Flagyl) B. Intravenous metronidazole C. Oral vancomycin (Vancocin) D. Intravenous vancomycin E. Rifaximin (Xifaxan)

Vancomycin, 125 mg orally 4 times daily for 10-14 days, is recommended for the first severe episode ofClostridium difficile colitis (SOR B). If the first episode is mild to moderate, oral metronidazole, 500 mg3 times daily for 10-14 days, would be preferred. Intravenous vancomycin is not effective in the treatment of colitis. Rifaximin is not well studied and is not recommended in any current guidelines.

Which one of the following medications used in the treatment of osteoporosis can also be used to treat the pain associated with acute and chronic vertebral compression fractures? (check one) A. Calcitonin-salmon (Miacalcin) B. Raloxifene (Evista) C. Risedronate (Actonel) D. Teriparatide (Forteo) E. Zoledronic acid (Reclast)

While all of the medications listed can be used to treat osteoporosis, only calcitonin-salmon is useful in the management of pain associated with acute or chronic vertebral fractures. Calcitonin is an antiresorptive agent that has been shown to decrease the risk of vertebral fractures, but it is not considered a first-line treatment for osteoporosis because there are more effective agents. However, it does have modest analgesic properties that make it useful in the treatment of the pain associated with vertebral fractures.

The key factors in diagnosing streptococcal pharyngitis

fever over 100.4°F, tonsillar exudates, anterior cervical lymphadenopathy, and absence of cough.


संबंधित स्टडी सेट्स

Automatic Transmission / Transaxle

View Set

Nursing Process (PREPU Questions) CHP 14 - ASSESSMENT

View Set

GASTROİNTESTİNAL SİSTEM HASTALIKLARININ TANISINDA ANAMNEZ VE FİZİK MUAYENE

View Set

ATI Book Mental Health Chapters 23, 29, 30, 32, 33

View Set

DOT Hazard Classes and Divisions

View Set

Mobility, thermoregulation, diversity quiz

View Set

Anatomy & Physiology- CH 20 The Lymphatic System and Immunity

View Set

The Enlightenment and the American Revolution

View Set

Cost of Capital and Capital Structure (chap 15-17)

View Set

Econ chapter 14 & 16, ManEcon - Chapter 14 quiz, quiz 4, ECON TEST 3

View Set

Chapter 20 - Assessment of Respiratory Function

View Set